Le equazioni di secondo grado

Le equazioni di secondo grado in una incognita

_images/immagini12.png

Consideriamo il seguente problema: “ in un triangolo rettangolo l’ipotenusa è più lunga del cateto minore di 4 cm, mentre l’altro cateto è più lungo del cateto minore di 2 cm. Si vogliono trovare le misure dei tre lati”.

Si può formalizzare il problema indicando con \(x\) la misura incognita del cateto minore. La lunghezza dell’ipotenusa sarà \(x + 4\) , mentre quella dell’altro cateto \(x + 2\) . Applicando il teorema di Pitagora si ha: \(x ^{2 } + ( x + 2 ) ^{2 } = ( x + 4 ) ^{2 }\) . Dopo aver effettuato i calcoli e aver portato tutti i termini a sinistra del predicato uguale abbiamo: \(x ^{2 } - 4 x - 12 = 0\) .

Questa è una equazione di secondo grado in una incognita in quanto la variabile \(x\) vi compare elevata al secondo grado.

Equazione di secondo grado
Si dice equazione di secondo grado , un’equazione del tipo: \(a x ^{2 } + b x + c = 0\) con \(a , b , c \in \mathbb{R} e a \neq 0\) . I valori \(a\) , \(b\) , \(c\) prendono il nome di coefficienti e, in particolare, \(c\) viene detto termine noto.

Un’equazione di secondo grado si definisce:

monomia quando il secondo e il terzo coefficiente sono nulli \(a x ^{2 } = 0\)

incompleta pura quando il secondo coefficiente è nullo \(a x ^{2 } + c = 0\) ;

incompleta spuria quando il terzo coefficiente è nullo \(a x ^{2 } + b x = 0\) ;

completa quando i tre coefficienti sono tutti diversi da zero \(a x ^{2 } + b x + c = 0\) .

Risoluzione di un’equazione di secondo grado pura

Il coefficiente della x è nullo e l’equazione si presenta nella forma: \(ax ^{2 } + c = 0\) .

Si risolve portando a secondo membro il termine noto e dividendo per il coefficiente di x 2:

\(a x ^{2 } + c = 0 \rightarrow a x ^{2 } = - c \rightarrow x ^{2 } = - \frac{c }{a } \rightarrow x _{1,2 } = \pm \sqrt{- \frac{c }{a } }\)

_images/immagini11.png

Esempi

  • \(4 x ^{2 } - 9 = 0\) risoluzione \(4 x ^{2 } = + 9 \rightarrow x ^{2 } = \frac{9 }{4 } \rightarrow x _{1,2 } = \pm \sqrt{\frac{9 }{4 } } \rightarrow x _{1 } = - \frac{3 }{2 } \vee x _{2 } = + \frac{3 }{2 }\)
  • \(4 x ^{2 } + 9 = 0\) risoluzione \(4 x ^{2 } + 9 = 0 \rightarrow x ^{2 } = - \frac{9 }{4 }\) .
  • L’equazione non ammette soluzioni reali in quanto il quadrato di un numero reale non è mai negativo.
_images/immagini11.png

In generale, le soluzioni dell’equazione incompleta pura \(ax ^{2 } + c = 0\) dipendono dal segno di \(- \frac{c }{a }\) :

  • se -c/a>0, ovvero se a e c sono discordi, l’equazione ammette due soluzioni reali e distinte \(x _{1 } = - \sqrt{- \frac{c }{a } } \vee x _{2 } = + \sqrt{- \frac{c }{a } }\)
  • se -c/a<0, ovvero se a e c sono concordi, l’equazione non ammette soluzioni reali ;
  • se -c/a=0, allora \(c = 0\) , l’equazione ha due soluzioni reali coincidenti nulle \(x _{1 } = x _{2 } = 0\) .

Risoluzione di un’equazione incompleta spuria

Un’equazione incompleta spuria si presenta nella forma: \(a x ^{2 } + b x = 0\) .

Per risolverla, si raccoglie a fattore comune la \(x\) ; precisamente \(x ( a x + b ) = 0\)

Applicando la legge di annullamento del prodotto si ottiene \(x _{1 } = 0\) oppure \(ax + b = 0\) da cui \(x _{2 } = - \frac{b }{a }\) .

Pertanto un’equazione di questo tipo ha sempre due soluzioni reali distinte di cui una nulla.

_images/immagini11.png

Esempi

  • \(2 x ^{2 } - 4 x = 0\)

    Raccogliendo a fattor comune si ha:

    \(2 x ( x - 2 ) = 0\)

    da cui, applicando la legge di annullamento del prodotto, segue

    \(2x = 0 \vee x - 2 = 0\) da cui \(x _{1 } = 0 \vee x _{2 } = 2\) .

  • \(x ^{2 } + x = 0\) . Raccogliendo \(x\) a fattore comune, si ha \(x ( x + 1 ) = 0\) , da cui, applicando la legge di annullamento del prodotto, segue \(x = 0 \vee x + 1 = 0\) da cui \(x _{1 } = 0 \vee x _{2 } = - 1\)

_images/immagini11.png

Risoluzione di un’equazione completa

Per risolvere l’equazione di secondo grado completa si applica una formula che si ottiene utilizzando il metodo del completamento del quadrato:

  • \(a x ^{2 } + b x + c = 0\) :
  • \(4 a ^{2 } x ^{2 } + 4 a b x + 4 a c = 0\) :si moltiplicano ambo i membri per \(4a\) ;
  • \(4 a ^{2 } x ^{2 } + 4 a b x + 4 a c + b ^{2 } = b ^{2 }\) :si aggiunge ad ambo i membri \(b ^{2 }\) ;
  • \(4 a ^{2 } x ^{2 } + 4 a b x + b ^{2 } = b ^{2 } - 4 a c\) :si porta 4ac a secondo membro;
  • \(( 2 a x + b ) ^{2 } = b ^{2 } - 4 a c\) :il primo membro risulta il quadrato di un binomio;
  • \(k = 2 a x + b\) :si sostituisce il binomio \(2ax + b\) con la la variabile \(k\) ;
  • \(k ^{2 } = b ^{2 } - 4 a c\) :l’equazione diventa un’equazione di secondo grado pura in k;
  • \(k _{1,2 } = \pm \sqrt{b ^{2 } - 4 a c }\) :si calcola le soluzioni in \(k\) ;
  • \(2 a x + b = \pm \sqrt{b ^{2 } - 4 a c }\) :al posto di \(k\) si mette il binomio \(2ax + b\) ;
  • \(2 a x = - b \pm \sqrt{b ^{2 } - 4 a c }\) :si separa il monomio con l’incognita
  • \(x _{1,2 } = \frac{- b \pm \sqrt{b ^{2 } - 4 a c } }{2 a }\) :si risolve l’equazione di primo grado rispetto alla \(x\) .

D a quanto ottenuto possiamo osservare:

  • che la soluzione si ottiene esclusivamente operando sui coefficienti dell’equazione;
  • che il valore dell’incognita si ottiene seguendo due calcoli: \(x _{1 } = \frac{- b - \sqrt{b ^{2 } - 4 ac } }{2 a } \vee x _{2 } = \frac{- b + \sqrt{b ^{2 } - 4 ac } }{2 a }\)
  • che nel calcolo è coinvolta l’estrazione di radice quadrata: l’espressione \(b ^{2 } - 4 ac\) prende il nome di discriminante e si è solito porre \(b ^{2 } - 4 ac = \Delta ( \text{ delta} )\)

Questa formula si può applicare anche ai tipi di equazioni incomplete che abbiamo già studiato.

La parola discriminante deriva dal verbo discrimen (divisione); in effetti, il \(\Delta\) permette di effettuare una distinzione tra la tipologia delle soluzioni di un’equazione di secondo grado.

Si possono infatti presentare tre casi:

  • Primo caso \(\Delta = b ^{2 } - 4 a c > 0\) il radicale \(\sqrt{\Delta }\) è un numero reale e l’equazione ammette due soluzioni reali e distinte : \(x _{1 } = \frac{- b - \sqrt{\Delta } }{2 a } \vee x _{2 } = \frac{- b + \sqrt{\Delta } }{2 a }\)
  • Secondo caso: \(\Delta = b ^{2 } - 4 a c = 0\) l’equazione ammette due radici reali e coincidenti: \(x _{1 } = x _{2 } = - \frac{b }{2 a }\)
  • Terzo caso: \(\Delta = b ^{2 } - 4 a c < 0\) l ’equazione non ammette soluzioni reali.

Riassumendo e schematizzando si ha per l’equazione \(a x ^{2 } + b x + c = 0\) completa con \(a \neq 0\) :

Tabella19
Discriminante Soluzioni
\(\Delta > 0\) Due soluzioni reali e distinte \(x _{1 } = \frac{- b - \sqrt{\Delta } }{2 a } \vee x _{2 } = \frac{- b + \sqrt{\Delta } }{2 a }\)
\(\Delta = 0\) Due soluzioni reali e coincidenti \(x _{1 } = x _{2 } = - \frac{b }{2 a }\)
\(\Delta < 0\) Nessuna soluzione reale \(I.S. = \varnothing\)
_images/immagini11.png

Esempi

  • \(3 x ^{2 } - 5 x + 2 = 0\) : \(a = + 3, b = - 5, c = + 2\) . Calcolo del discriminante: \(\Delta = b ^{2 } - 4 ac = ( - 5 ) ^{2 } - 4 ( + 3 ) ( + 2 ) = 25 - 24 = 1\) . Poiché \(\Delta > 0\) l’equazione ammette due soluzioni reali e distinte: \(x _{1,2 } = \frac{- b \pm \sqrt{\Delta } }{2 a } \rightarrow x _{1,2 } = \frac{- ( - 5 ) \pm \sqrt{1 } }{( + 2 ) ( + 3 ) } \rightarrow x _{1,2 } = \frac{5 \pm 1 }{6 } \rightarrow x _{1 } = 1 \vee x _{2 } = \frac{2 }{3 }\)
  • \(4 x ^{2 } - 12 x + 9 = 0\) : \(a = + 4, b = - 12, c = + 9\) : Calcolo del discriminante: \(\Delta = ( - 12 ) ^{2 } - 4 ( + 4 ) ( + 9 ) = 144 - 144 = 0\) . Poiché \(\Delta = 0\) l’equazione ammette due soluzioni reali coincidenti \(x _{1,2 } = - \frac{b }{2 a } \rightarrow x _{1,2 } = \frac{- ( - 12 ) }{2 ( + 4 ) } = \frac{12 }{8 } \rightarrow x _{1 } = x _{2 } = \frac{3 }{2 }\) .
  • \(x ^{2 } - x + 3 = 0\) : \(a = + 1, b = - 1, c = + 3\) . Calcolo del discriminante: \(\Delta = ( - 1 ) ^{2 } - 4 ( + 1 ) ( + 3 ) = 1 - 12 = - 11\) . Poiché \(\Delta < 0\) l’equazione non ammette soluzioni reali.
_images/immagini11.png

Formula ridotta per equazioni di secondo grado

Se nell’equazione \(a x ^{2 } + b x + c = 0\) il coefficiente b è un numero pari, conviene applicare una formula, detta formula ridotta, che semplifica i calcoli.

Supponiamo \(b = 2 k\) , l’equazione \(a x ^{2 } + b x + c = 0\) diventa \(a x ^{2 } + 2 k x + c = 0\) nella formula risolutiva dell’equazione si ottiene:

\(\begin{array}{l } x _{1,2 } = \frac{- 2 k \pm \sqrt{( 2 k ) ^{2 } - 4 a c } }{2 a } = \frac{- 2 k \pm \sqrt{4 k ^{2 } - 4 a c } }{2 a } = \frac{- 2 k \pm \sqrt{4 ( k ^{2 } - a c ) } }{2 a } = \\ = \frac{- 2 k \pm 2 \sqrt{k ^{2 } - a c } }{2 a } = \frac{2 \left( - k \pm \sqrt{k ^{2 } - a c } \right) }{2 a } = \frac{- k \pm \sqrt{k ^{2 } - a c } }{a } \end{array}\)

Dato che \(b = 2 k\) quindi \(k = \frac{b }{2 }\) la formula ridotta che conviene utilizzare quando b è pari è: \(x _{1,2 } = \frac{\left( - \frac{b }{2 } \right) \pm \sqrt{\left( \frac{b }{2 } \right) ^{2 } - a c } }{a }\)

La quantità sotto radice, uguale a \(\frac{\Delta }{4 }\) , è detta anche discriminante ridotto .

Vediamo qualche applicazione pratica della formula ridotta.

_images/immagini11.png

Esempi

  • \(x ^{2 } - 4 x + 3 = 0\)

    Il coefficiente di primo grado è pari, per cui conviene utilizzare la

    formula ridotta \(x _{1,2 } = \frac{\left( - \frac{b }{2 } \right) \pm \sqrt{\left( \frac{b }{2 } \right) ^{2 } - a c } }{a } = \frac{- ( - 2 ) \pm \sqrt{( - 2 ) ^{2 } - 1 ( 3 ) } }{1 } = 2 \pm \sqrt{1 }\) quindi \(x _{1 } = 1 \vee x _{2 } = 3\) .

  • \(- x ^{2 } - 2 x + 24 = 0\) \(x _{1,2 } = \frac{\left( - \frac{b }{2 } \right) \pm \sqrt{\left( \frac{b }{2 } \right) ^{2 } - a c } }{a } = \frac{- ( - 1 ) \pm \sqrt{( - 1 ) ^{2 } - ( - 1 ) ( 24 ) } }{- 1 } = - 1 \pm \sqrt{25 }\) quindi \(x _{1 } = - 6 \vee x _{2 } = 4\)

  • \(- 3 x ^{2 } - 6 x + 12 = 0\)

    d ividendo l’equazione per

    \(- 3\) , per il secondo principio di equivalenza, si ha l’equazione equivalente \(x ^{2 } + 2 x - 4 = 0\) Poiché il coefficien t e della \(x\) è pari si può applicare la formula ridotta. \(x _{1,2 } = \frac{\left( - \frac{b }{2 } \right) \pm \sqrt{\left( \frac{b }{2 } \right) ^{2 } - a c } }{a } = - 1 \pm \sqrt{1 + 4 } = - 1 \pm \sqrt{5 }\) quindi \(x _{1 } = - 1 + \sqrt{5 } \vee x _{2 } = - 1 - \sqrt{5 }\)

_images/immagini11.png

Quando \(b\) è pari e \(a = 1\) , la formula si dice ridottissima \(x _{1,2 } = \left( - \frac{b }{2 } \right) \pm \sqrt{\left( \frac{b }{2 } \right) ^{2 } - c }\) .

  • \(x ^{2 } - 6 x + 8 = 0\) \(x _{1,2 } = \left( - \frac{b }{2 } \right) \pm \sqrt{\left( \frac{b }{2 } \right) ^{2 } - c } = 3 \pm \sqrt{9 - 8 } = 3 \pm 1 \rightarrow x _{1 } = 2 ; x _{2 } = 4\) .

Riassumiamo e schematizziamo la risoluzione di un’equazione di secondo grado:

Equazioni incomplete

Tabella1
Coefficienti Nome Equazione Soluzioni
\(b = 0, c = 0\) Monomia \(a x ^{2 } = 0\) \(x _{1 } = x _{2 } = 0\)
\(b = 0, c \neq 0\) Pura \(a x ^{2 } + c = 0\) se a e c sono concordi \(I.S. = \varnothing\) se a e c sono discordi \(x _{1 } = + \sqrt{- \frac{c }{a } } \vee x _{2 } = - \sqrt{- \frac{c }{a } }\)
\(b \neq 0, c = 0\) Spuria \(a x ^{2 } + b x = 0\) \(x _{1 } = 0 \vee x _{2 } = - \frac{b }{a }\)

Equazione completa \(a x ^{2 } + b x + c = 0\) con \(a \neq 0\)

Tabella2
Discriminante numero soluzioni soluzioni
\(\Delta > 0\) Due soluzioni reali e distinte \(x _{1 } = \frac{- b - \sqrt{b ^{2 } - 4 ac } }{2 a } \vee x _{2 } = \frac{- b + \sqrt{b ^{2 } - 4 ac } }{2 a }\)
\(\Delta = 0\) Due soluzioni reali e coincidenti \(x _{1 } = x _{2 } = - \frac{b }{2 a }\)
\(\Delta < 0\) Nessuna soluzione reale \(I.S. = \varnothing\)

Equazioni che si possono risolvere con opportune sostituzioni

_images/immagini11.png

Esempi

  • \(( x - 1 ) ^{2 } = 16\) Sostituendo \(x - 1 = t\) l’equazione diventa \(t ^{2 } = 16\) , le cui soluzioni sono \(t _{1 } = - 4 \vee t _{2 } = + 4\) . Per determinare la x sostituiamo i valori di t trovati nella relazione \(x - 1 = t\) si ha \(x - 1 = - 4 \vee x - 1 = + 4\) quindi l’equazione assegnata ammette le due soluzioni \(x _{1 } = - 3 \vee x _{2 } = 5\) .
_images/immagini11.png
  • \(( x - 1 ) ^{2 } + 2 ( x - 1 ) = 0\) Sostituendo \(x - 1 = t\) l’equazione diventa \(t ^{2 } + 2t = 0\) le cui soluzioni sono \(t ( t + 2 ) = 0 \rightarrow t _{1 } = 0 \vee t + 2 = 0 \rightarrow t _{2 } = - 2\) . Sostituendo \(x - 1 = t\) si ha \(x - 1 = 0 \vee x - 1 = - 2\) quindi l’equazione assegnata ammette le due soluzioni \(x _{1 } = - 1 \vee x _{2 } = 1\) .

Discussione e risoluzione di equazioni numeriche frazionarie

Un’equazione in cui compare l’incognita al denominatore si chiama frazionaria o fratta.

_images/immagini11.png

Esempi

  • \(\frac{3 x + 2 }{1 + x } = \frac{2 x + 3 }{x - 2 }\)

    1° passo: determiniamo il m.c.m. dei denominatori: \(m.c.m. = ( 1 + x ) \cdot ( x - 2 )\)

    2° passo: imponiamo le Condizioni di Esistenza (CE): \(C.E. x \neq - 1 \wedge x \neq 2\) La ricerca dei valori che risolvono l’equazione si restringe ai numeri reali appartenenti all’insieme, \(D = R – \left \{ - 1, \right .2 \}\) detto Dominio dell’equazione o Insieme di Definizione (abbreviato ID).

    3° passo: applichiamo il primo principio d’equivalenza trasportando al primo membro la frazione del secondo membro \(\frac{3 x + 2 }{1 + x } - \frac{2 x + 3 }{x - 2 } = 0\) . Riduciamo allo stesso denominatore (m.c.m.) \(\frac{( 3 x + 2 ) \cdot ( x - 2 ) - ( 2 x + 3 ) \cdot ( 1 + x ) }{( 1 + x ) \cdot ( x - 2 ) } = 0\)

    4° passo: applichiamo il secondo principio moltiplicando ambo i membri per il m.c.m., certamente diverso da zero per le condizioni poste; l’equazione diventa: \(( 3 x + 2 ) \cdot ( x - 2 ) - ( 2 x + 3 ) \cdot ( 1 + x ) = 0\)

    5° passo: svolgendo i calcoli ci accorgiamo che l’equazione è di secondo grado; portiamo l’equazione alla forma canonica: \(3 x ^{2 } - 6 x + 2 x - 4 - 2 x - 3 - 2 x ^{2 } - 3 x = 0 \rightarrow x ^{2 } - 9 x - 7 = 0\)

    6° passo: calcoliamo il discriminante: \(\Delta = b ^{2 } - 4 a c = 81 + 28 = 109\) essendo positivo, l’equazione è determinata e ammette due soluzioni reali distinte: \(x _{1,2 } = \frac{9 \pm \sqrt{109 } }{2 } \rightarrow x _{1 } = \frac{9 - \sqrt{109 } }{2 } \vee x _{2 } = \frac{9 + \sqrt{109 } }{2 }\)

    7° passo: confrontiamo le soluzioni con le C.E. ; in questo caso le radici appartengono all’insieme D; diciamo che sono accettabili e l’insieme soluzione è: \(\mathbf{I.S. } = \left\{ \frac{9 - \sqrt{109 } }{2 } , \frac{9 + \sqrt{109 } }{2 } \right\}\)

  • \(\frac{x ^{2 } }{x ^{2 } - 3 x + 2 } = \frac{x - 2 }{x - 1 } + \frac{1 }{x + 2 }\)

    1° passo: determiniamo il m.c.m. dei denominatori; per fare questo dobbiamo scomporre in fattori i denominatori. Riscriviamo: \(\frac{x ^{2 } }{( x - 2 ) ( x - 1 ) } = \frac{x - 2 }{x - 1 } + \frac{1 }{x + 2 }\) il m.c.m. è \(( x - 2 ) ( x - 1 ) ( x + 2 )\)

    2° passo: imponiamo le Condizioni di Esistenza: \(C.E. x \neq 1 \text{ None} \wedge x \neq 2 \text{ None} \wedge x \neq - 2\) quindi \(D = \mathbb{R} – \{ 1, 2, - 2 \} = I.D.\) .

    3° passo: trasportiamo al primo membro ed uguagliamo a zero; riduciamo allo stesso denominatore (m.c.m.) ambo i membri dell’equazione: \(\frac{x ^{3 } + 2 x ^{2 } - x ^{2 } + 3 x - 2 - x ^{3 } - 2 x ^{2 } + 4 x ^{2 } + 8 x - 4 x - 8 }{( x - 2 ) ( x - 1 ) ( x + 2 ) } = 0\)

    4° passo: applichiamo il secondo principio di equivalenza moltiplicando ambo i membri per il m.c.m., certamente diverso da zero per le condizioni poste; l’equazione diventa: \(3 x ^{2 } + 7 x - 10 = 0\)

    5° passo: calcoliamo il discriminante: \(\Delta = b ^{2 } - 4 a c = 49 + 120 = 169\) essendo positivo, l’equazione è determinata e ammette due soluzioni reali distinte: \(x _{1,2 } = \frac{- 7 \pm 13 }{6 } \rightarrow x _{1 } = - \frac{10 }{3 } \vee x _{2 } = 1\)

    6° passo: confrontiamo con le C.E. ; in questo caso solo \(x _{1 }\) appartiene all’insieme D; diciamo che l’insieme soluzione è: \(\mathbf{I.S. } = \left\{ - \frac{10 }{3 } \right\}\) mentre \(x _{2 } = 1\) non è accettabile.

_images/immagini11.png

Discussione e risoluzione di equazioni letterali

Ricordiamo la:

Una equazione è letterale se i coefficienti dell’incognita sono espressioni letterali, cioè se oltre all’incognita (in genere indicata con la lettera x) compare un’altra lettera (in genere a, b, k, …) detta parametro.

_images/immagini11.png

Esempi

  • Discutere al variare di k la realtà delle soluzioni * *dell’equazione \(k x ^{2 } - ( 2 k - 1 ) x + ( k - 3 ) = 0\) .

    L’equazione è letterale di secondo grado nell’incognita \(x\) , i cui coefficienti dipendono dal parametro \(k\) . Il parametro \(k\) può assumere qualunque valore numerico e l’equazione rappresenta una famiglia di equazioni le cui caratteristiche variano a seconda dei valori attribuiti al parametro. Notiamo subito che se \(k\) assume il valore zero, l’equazione non è più di secondo grado. Se \(k\) assume il valore 3, l’equazione è ancora di secondo grado ma è incompleta (spuria) perché priva del termine noto.

    Discutere un’equazione letterale di secondo grado significa analizzare come varia il suo insieme delle soluzioni al variare del parametro.

    Ricordando la formula \(x _{1,2 } = \frac{- b \pm \sqrt{b ^{2 } - 4 a c } }{2 a }\) in cui compaiono i tre coefficienti a, b, c possiamo dire

    • il primo coefficiente è k, se \(k = 0\) l’equazione diventa \(x - 3 = 0\) di primo grado con \(I.S. = \{ 3 \}\) ;
    • il secondo coefficiente è -2k+1, se è nullo, ossia se \(k = \frac{1 }{2 }\) l’equazione diventa \(\frac{1 }{2 } x ^{2 } - \frac{5 }{2 } = 0\) equazione pura con due soluzioni reali opposte \(x _{1 } = - \sqrt{5 } \vee x _{2 } = \sqrt{5 }\) ;
    • il terzo coefficiente è k-3, se è nullo, cioè se \(k = 3\) l’equazione diventa \(3 x ^{2 } - 5 x = 0\) , equazione spuria con due soluzioni reali \(x _{1 } = 0 \vee x _{2 } = \frac{5 }{3 }\)

P er tutti i valori di \(k\) dell’insieme \(\mathbb{R} - \left\{ 0, \frac{1 }{2 } ,3 \right\}\) l’equazione è completa, l’esistenza di soluzioni reali dipende dal discriminante \(\Delta = \left( - 2 k + 1 \right) ^{2 } - 4 k \left( k - 3 \right) = 8 k + 1\) , quindi

se \(8 k + 1 < 0 \rightarrow k < - \frac{1 }{8 }\) l’equazione non ammette soluzioni reali e \(I.S. = \varnothing\) ;

se \(8 k + 1 \geq 0 \rightarrow k \geq - \frac{1 }{8 }\) l’equazione ammette due soluzioni reali:

  • distinte se \(k > - \frac{1 }{8 } \rightarrow x _{1,2 } = \frac{\left( 2 k - 1 \right) \pm \sqrt{8 k + 1 } }{2 k }\)
  • coincidenti se \(k = - \frac{1 }{8 } \rightarrow x _{1 } = x _{2 } = 5\)

Riassumendo, data l’equazione: \(k x ^{2 } - ( 2 k - 1 ) x + ( k - 3 ) = 0 con k \in \mathbb{R}\)

Tabella4
Condizioni sul parametro Insieme soluzione Equazione
\(k = 0\) \(x = 3\) Di primo grado
\(k = \frac{1 }{2 }\) \(x _{1 } = - \sqrt{5 } \vee x _{2 } = + \sqrt{5 }\) Pura
\(k = 3\) \(x _{1 } = 0 \vee x _{2 } = \frac{5 }{3 }\) Spuria
\(k \in \mathbb{R} - \left\{ 0, \frac{1 }{2 } ,3 \right\}\) \(\Delta = 8 k + 1\) Completa:
\(k < - \frac{1 }{8 }\) \(\Delta < 0\) non esistono soluzioni reali \(I.S. = \varnothing\)
\(k \geq - \frac{1 }{8 }\) \(\Delta \geq 0\) esistono soluzioni reali
\(k > - \frac{1 }{8 }\) \(x _{1 } = \frac{\left( 2 k - 1 \right) - \sqrt{8 k + 1 } }{2 k } \vee x _{2 } = \frac{\left( 2 k - 1 \right) + \sqrt{8 k + 1 } }{2 k }\) reali distinte
\(k = - \frac{1 }{8 }\) \(x _{1 } = x _{2 } = 5\) reali coincidenti
  • Discutere, al variare di * :math:`k in mathbb{R}` * la realtà delle radici dell’equazione * :math:`x ^{2 } - 3 x + 1 - k = 0` . Il primo e il secondo coefficiente non dipendono dal parametro *k , quindi analizziamo il terzo coefficiente. S e \(k = 1\) l’equazione diventa un’equazione spuria con due radici reali \(x _{1 } = 0 \vee x _{2 } = 3\) . Per tutti i valori di k dell’insieme \(\mathbb{R} - \{ 1 \}\) l’equazione è completa, l’esistenza di soluzioni reali dipende dal discriminante \(\Delta = 9 - 4 ( 1 - k ) = 4 k + 5\) , quindi:

se \(k < - \frac{5 }{4 }\) l’equazione non ammette soluzioni reali e \(I.S. = \varnothing\)

se \(k \geq - \frac{5 }{4 }\) l’equazione ammette due radici reali

  • distinte se \(k > - \frac{5 }{4 } \rightarrow x _{1 } = \frac{3 - \sqrt{4 k + 5 } }{2 } \vee x _{2 } = \frac{3 + \sqrt{4 k + 5 } }{2 }\)
  • coincidenti se \(k = - \frac{5 }{4 } \rightarrow x _{1 } = x _{2 } = \frac{3 }{2 }\)

Riassumendo data l’equazione \(x ^{2 } - 3 x + 1 - k = 0 con k \in \mathbb{R}\) :

Tabella8
Condizioni sul parametro Insieme soluzione Equazione
\(k = 1\) \(x _{1 } = 0 \vee x _{2 } = 3\) Spuria
\(k \in \mathbb{R} - \left\{ 1 \right\}\) \(\Delta = 4 k + 5\) Completa:
\(k < - \frac{5 }{4 }\) \(\Delta < 0\) \(I.S. = \varnothing\) non esistono soluzioni reali
\(k \geq - \frac{5 }{4 }\) esistono soluzioni reali \(\Delta \geq 0\)
\(k > - \frac{5 }{4 }\) reali distinte \(x _{1 } = \frac{3 - \sqrt{4 k + 5 } }{2 } \vee x _{2 } = \frac{3 + \sqrt{4 k + 5 } }{2 }\)
\(k = - \frac{5 }{4 }\) reali coincidenti \(x _{1 } = x _{2 } = \frac{3 }{2 }\)
  • *Discutere la seguente equazione letterale: * \(\frac{x ^{2 } }{m - 1 } + 3 + m = \frac{2 m x }{m - 1 } \left( 1 + \frac{1 }{m } \right)\) L’equazione pur presentando delle frazioni è intera, in quanto l’incognita \(x\) non compare al denominatore. Se \(m = 0\) oppure \(m = 1\) l’equazione è priva di significato, quindi \(\mathbf{C.E. } m \neq 0 \text{ None} \wedge m \neq 1\) .
    • Trasportiamo a sinistra del segno di uguaglianza i termini di destra ed eseguiamo il calcolo nella parentesi: \(\frac{x ^{2 } }{m - 1 } + 3 + m - \frac{2 m x }{m - 1 } \left( 1 + \frac{1 }{m } \right) = 0 \rightarrow \frac{x ^{2 } }{m - 1 } + 3 + m - \frac{2 m x }{m - 1 } - \frac{2 m x }{m - 1 } \cdot \frac{1 }{m } = 0\) ;
    • Semplifichiamo \(m\) nell’ultimo termine, poiché nelle C.E. \(m \neq 0\) si ha \(\frac{x ^{2 } }{m - 1 } + 3 + m - \frac{2 mx }{m - 1 } - \frac{2 x }{m - 1 }\) ;
    • Riduciamo allo stesso denominatore \(m - 1\) , eliminiamo il denominatore essendo \(m \neq 1\) per le C.E. s i ha: \(x ^{2 } + 3 m - 3 + m ^{2 } - m - 2 m x - 2 x = 0\) ;
    • Scriviamo l’equazione di secondo grado in forma canonica \(x ^{2 } - 2 x ( m + 1 ) + m ^{2 } + 2 m - 3 = 0\) Discussione
      • il primo coefficiente \(a = 1\) non dipende dal valore del parametro, quindi l’equazione è di secondo grado per qualunque valore di \(m \in \mathbb{R} - \{ 0, 1 \}\) ;
      • il secondo coefficiente \(b = - 2 ( m + 1 )\) : se \(m = - 1\) l’equazione diventa \(x ^{2 } - 4 = 0\) , equazione pura con due soluzioni reali opposte \(x _{1 } = - 2 \vee x _{2 } = 2\) ;
      • il terzo coefficiente \(c = m ^{2 } + 2 m - 3\) : se \(c = m ^{2 } + 2 m - 3 = 0 \rightarrow m = 1 \vee m = - 3\) (non consideriamo il caso \(m = 1\) per le C.E.) l’equazione diventa \(x ^{2 } + 4 x = 0\) , equazione spuria con due soluzioni reali \(x _{1 } = 0 \vee x _{2 } = - 4\) .

Prima conclusione: per tutti i valori di m nell’insieme \(\mathbb{R} - \{ 0,1 , - 1, - 3 \}\) l’equazione è completa e l’esistenza di soluzioni reali dipende dal discriminante.

  • Calcoliamo il discriminante: \(\frac{\Delta }{4 } = ( m + 1 ) ^{2 } - ( m ^{2 } + 2 m - 3 ) = 4\) ; esso risulta indipendente dal valore del parametro e sempre positivo, quindi l’equazione ammette due soluzioni reali distinte \(x _{1 } = m - 1 \vee x _{2 } = m + 3\) .

Riassumendo in una tabella tutti i risultati ottenuti data l’equazione \(\frac{x ^{2 } }{m - 1 } + 3 + m = \frac{2 m x }{m - 1 } \left( 1 + \frac{1 }{m } \right) con m \in \mathbb{R}\) :

Tabella5
Condizioni sul parametro Insieme soluzione Equazione
\(m = 0 \vee m = 1\)   Priva di significato
\(m = - 1\) \(x _{1 } = - 2 \vee x _{2 } = 2\) Pura
\(m = - 3\) \(x _{1 } = 0 \vee x _{2 } = - 4\) Spuria
\(m \in \mathbb{R} - \{ 0,1 , - 1, - 3 \}\) \(x _{1 } = m - 1 \vee x _{2 } = m + 3\) Completa: \(\Delta = 4\)
  • Discutere la seguente equazione parametrica: \(\frac{k + x }{2 x } \left( \frac{k + x }{k - x } + \frac{k - x }{k + x } \right) = k + \frac{2 k }{k x - x ^{2 } } - 1\) L’equazione è fratta, poiché nel denominatore compare l’incognita x.
    • Trasportiamo i termini del secondo membro a sinistra del segno uguale e scomponiamo in fattori i denominatori: \(\frac{k + x }{2 x } \left( \frac{k + x }{k - x } + \frac{k - x }{k + x } \right) - k - \frac{2 k }{x ( k - x ) } + 1 = 0\) ; \(C.E. x \neq 0 \text{ None} \wedge x \neq k \text{ None} \wedge x \neq - k\)
    • Svolgiamo i calcoli nella parentesi e moltiplichiamo \(\frac{k ^{2 } + x ^{2 } }{x ( k - x ) } - k - \frac{2 k }{x ( k - x ) } + 1 = 0\) ;
    • Riduciamo allo stesso denominatore ed eliminiamo il denominatore \(k x ^{2 } + k x \cdot ( 1 - k ) + k \cdot ( k - 2 ) = 0\) ;
    • il primo coefficiente è k, se \(k = 0\) le C.E. si riducono a \(x \neq 0\) e l’equazione diventa \(0x = 0\) indeterminata, quindi \(I.S. = \mathbb{R} - \{ 0 \}\) per le condizioni poste sull’incognita. Avendo studiato il caso k=0, possiamo ora suppore \(k \neq 0\) , dividiamo tutti i coefficienti per k , l’equazione diventa \(x ^{2 } + x \cdot ( 1 - k ) + ( k - 2 ) = 0\) ;
    • Il secondo coefficiente è 1-k, se \(k = 1\) le C.E. sono \(x \neq 0 \text{ None} \wedge x \neq 1 \text{ None} \wedge x \neq - 1\) e l’equazione diventa \(x ^{2 } - 1 = 0\) , le soluzioni sono \(x _{1 } = - 1 \vee x _{2 } = 1\) che non sono accettabili per le C.E.
    • il terzo coefficiente è k-2, se \(k = 2\) le C.E. sono \(x \neq 0 \text{ None} \wedge x \neq 2 \text{ None} \wedge x \neq - 2\) e l’equazione diventa \(x ^{2 } - x = 0\) le cui soluzioni sono \(x _{1 } = 0 \vee x _{2 } = 1\) di cui \(x _{1 } = 0\) non accettabile per le C.E.

Per \(k \in \mathbb{R} - \{ 0,1 ,2 \}\) l’equazione è completa, l’esistenza di soluzioni reali dipende dal discriminante \(\Delta = ( 1 - k ) ^{2 } - 4 ( k - 2 ) = ( k - 3 ) ^{2 }\) ; essendo \(\Delta \geq 0 \forall k\) , si avranno sempre due soluzioni reali.

  • coincidenti se \(k = 3 \rightarrow x _{1 } = x _{2 } = 1\) accettabili essendo le \(C.E. x \neq - 3 \text{ None} \wedge x \neq 0 \text{ None} \wedge x \neq 3\) ;
  • distinte se \(k \neq 3 \rightarrow x _{1 } = 1 \vee x _{2 } = k - 2\) e confrontando con le C.E. si ottiene \(x _{1 } = 1\) non accettabile se \(k = - 1\) ; \(x _{2 }\) sempre accettabile per \(k \in \mathbb{R} - \{ 0,1 ,2 ,3 , - 1 \}\) .

Riassumendo data l’equazione \(\frac{k + x }{2 x } \left( \frac{k + x }{k - x } + \frac{k - x }{k + x } \right) = k + \frac{2 k }{k x - x ^{2 } } - 1 con k \in \mathbb{R}\) :

Tabella7
Condizioni sul parametro Condizioni sull’incognita Insieme Soluzione Equazione
  \(x \neq - k \text{ None} \wedge x \neq 0 \text{ None} \wedge x \neq k\)    
\(k = 0\) \(x \neq 0\) \(I.S. = \mathbb{R} - \{ 0 \}\) indeterminata
\(k = 1\) \(x \neq - 1 \text{ None} \wedge x \neq 0 \text{ None} \wedge x \neq 1\) \(x _{1 } = - 1 \vee x _{2 } = 1\) non accet. pura
\(k = 2\) \(x \neq - 2 \text{ None} \wedge x \neq 0 \text{ None} \wedge x \neq 2\) \(x _{1 } = 0 \vee x _{2 } = 1\) \(x _{1 }\) non accettabile spuria
\(k \in \mathbb{R} - \left \{ \right .0,1 ,2 \}\)     Completa \(\Delta = ( k - 3 ) ^{2 }\)
\(k = 3\) \(x \neq - 3 \text{ None} \wedge x \neq 0 \text{ None} \wedge x \neq 3\) \(x _{1 } = x _{2 } = 1\) accettabili
\(k \in \mathbb{R} - \left \{ \right .0,1 ,2 ,3 \}\) \(x \neq - k \text{ None} \wedge x \neq 0 \text{ None} \wedge x \neq k\) \(x _{1 } = 1 \vee x _{2 } = k - 2\)  
\(k = - 1\)   \(x _{1 } = 1\) non accettabile  
\(k \in \mathbb{R} - \left \{ \right .0,1 ,2 ,3 , - 1 \}\)   \(x _{2 } = k - 2\) accettabile  

Relazioni tra soluzioni e coefficienti

Consideriamo una generica equazione di secondo grado \(a x ^{2 } + b x + c = 0\) nell’ipotesi in cui ammetta soluzioni reali (cioè \(\Delta \geq 0\) ), sommiamo e moltiplichiamo le soluzioni (o radici) dell’equazione:

\(x _{1 } + x _{2 } = \frac{- b - \sqrt{\Delta } }{2 a } + \frac{- b + \sqrt{\Delta } }{2 a } = - \frac{2 b }{2 a } = - \frac{b }{a }\)

\(x _{1 } \cdot x _{2 } = \left( \frac{- b - \sqrt{\Delta } }{2 a } \right) \cdot \left( \frac{- b + \sqrt{\Delta } }{2 a } \right) = - \frac{b ^{2 } - \Delta }{2 a } = \frac{b ^{2 } + 4 a c - b ^{2 } }{4 a ^{2 } } = \frac{4 a c }{4 a ^{2 } } = \frac{c }{a }\)

Quindi, la somma delle radici è \(x _{1 } + x _{2 } = - \frac{b }{a }\) , il prodotto delle radici è \(x _{1 } \cdot x _{2 } = \frac{c }{a }\) .

Osserviamo che queste relazioni tra radici e coefficienti dell’equazione valgono anche nel caso in cui le radici non siano reali ( Δ<0).

_images/immagini11.png

Esempi

  • Determina la somma e il prodotto delle soluzioni dell’equazione \(2 x ^{2 } + 11 x - 3 = 0\) senza risolverla. Calcolo il discriminante \(\Delta = 145 > 0\) pertanto le radici sono reali e distinte. Applicando le precedenti formule si ha: \(x _{1 } + x _{2 } = - \frac{11 }{2 }\) ; \(x _{1 } \cdot x _{2 } = - \frac{3 }{2 }\) .
  • Data l’equazione : \(x ^{2 } \sqrt{2 } + 3 x - 2 \sqrt{2 } = 0\) , determina, senza risolverla, la somma e il prodotto delle radici. Calcolo il discriminante \(\Delta = 25 > 0\) pertanto le radici sono reali e distinte. Applicando le precedenti formule si ha: \(x _{1 } + x _{2 } = - \frac{3 }{\sqrt{2 } } = - \frac{3 \sqrt{2 } }{2 }\) ; \(x _{1 } \cdot x _{2 } = - \frac{2 \sqrt{2 } }{\sqrt{2 } } = - 2\) .
  • Determina somma e prodotto delle radici dell’equazione: \(x ^{2 } + 2 x + 15 = 0\) Calcolo il discriminante \(\Delta = - 56 < 0\) le radici non sono reali anche se la loro somma e il loro prodotto sono reali, infatti applicando le precendeti precedenti formule si ha: \(x _{1 } + x _{2 } = - 2\) e \(x _{1 } \cdot x _{2 } = 15\) .
  • Determina le radici dell’equazione \(x ^{2 } + 2 x - 15 = 0\) senza applicare la formula risolutiva, ma sfruttando la somma e il prodotto delle radici stesse. Calcolo il discriminante \(\Delta = 64\) pertanto le radici sono reali. Esse hanno come somma \(- \frac{b }{a } = – 2\) e come prodotto \(\frac{c }{a } = – 15\) . Le coppie di numeri che hanno per prodotto -15 sono -3 e +5, oppure +3 e -5, oppure +15 e -1, oppure -15 e +1. Tra tutte queste coppie l’unica che ha per somma -2 è la coppia -5 e +3. Pertanto le soluzioni dell’equazione sono \(x _{1 } = 3 \vee x _{2 } = - 5\) .
  • Determina le radici dell’equazione \(x ^{2 } - 12 x + 36 = 0\) senza applicare la formula risolutiva, ma sfruttando la somma e il prodotto delle radici stesse. Il discriminate \(\Delta = 12 ^{2 } - 4 \cdot 36 = 144 - 144 = 0\) . Le radici sono coincidenti, applicando la formula risolutiva si ha \(x _{1 } = x _{2 } = 6\) . Applicando le formule per calcolare somma e prodotto si ha \(x _{1 } + x _{2 } = 12\) e \(x _{1 } \cdot x _{2 } = 36\) da cui si conclude ugualmente che \(x _{1 } = x _{2 } = 6\) .
  • Si determini la relazione che lega i coefficienti della generica equazione di secondo grado alla somma dei reciproci delle radici. Si vuole cioè esprimere \(\frac{1 }{x _{1 } } + \frac{1 }{x _{2 } }\) attraverso i coefficienti a, b, c dell’equazione. Osserviamo in via preliminare che tale somma è possibile con la condizione \(x _{1 } \neq 0 \text{ None} \wedge x _{2 } \neq 0\) che implica \(c \neq 0\) . Si ha: \(\frac{1 }{x _{1 } } + \frac{1 }{x _{2 } } = \frac{x _{2 } + x _{1 } }{x _{1 } \cdot x _{2 } } = \frac{- \frac{b }{a } }{\frac{c }{a } } = - \frac{b }{c }\)
  • Si determini la relazione che lega i coefficienti della generica equazione di secondo grado alla differenza delle radici. Poiché non abbiamo informazioni a priori su quale delle due soluzioni sia la maggiore, calcoliamo il valore assoluto della differenza richiesta il calcolo diventa \(\left\lvert x _{1 } - x _{2 } \right\rvert = \left\lvert \frac{- b - \sqrt{\Delta } }{2 a } - \frac{- b + \sqrt{\Delta } }{2 a } \right\rvert = \left\lvert - \frac{2 \sqrt{\Delta } }{2 a } \right\rvert = \left\lvert - \frac{\sqrt{\Delta } }{a } \right\rvert\) .
_images/immagini11.png

Determinare due numeri conoscendone la somma e il prodotto

Consideriamo la generica equazione di secondo grado \(a x ^{2 } + bx + c = 0\) nell’ipotesi in cui ammetta soluzioni reali \(x _{1 }\) e \(x _{2 }\) . Essendo \(a \neq 0\) , è possibile dividere ambo i membri per a , ottenendo: \(x ^{2 } + \frac{b }{a } x + \frac{c }{a } = 0 \text{ }\) . Dato che \(s = x _{1 } + x _{2 } = - \frac{b }{a }\) e \(p = x _{1 } \cdot x _{2 } = \frac{c }{a }\) si avrà \(x ^{2 } - s x + p = 0\) .

Tale equazione risolve quindi la classe di problemi del tipo: “determinare due numeri che sommati danno s * *e moltiplicati danno p.

Dall’equazione \(x ^{2 } - s x + p = 0\) discende che tali numeri esistono reali se e solo se \(\Delta = s ^{2 } - 4 p \geq 0\) ovvero se il quadrato della somma è maggiore o uguale al quadruplo del loro prodotto.

_images/immagini11.png

Esempi

  • Determinare due numeri che sommati danno 12 e moltiplicati danno 35. L’equazione che risolve il problema è: \(x ^{2 } - 12 x + 35 = 0\) . Le soluzioni sono \(x _{1 } = 5 \vee x _{2 } = 7\) .
  • Determinare due numeri che sommati danno 5 e moltiplicati danno 9. L’equazione che risolve il problema è: \(x ^{2 } - 5 x + 9 = 0\) . Poiché \(\Delta = s ^{2 } - 4 p = 25 - 36 = - 11\) , l’equazione non ammette soluzioni reali e, di conseguenza, non esistono due numeri reali aventi la somma e il prodotto richiesti.
_images/immagini11.png

Problemi di natura geometrica di secondo grado

Problema

Determinate la misura della diagonale di un rettangolo avente il perimetro di 80m. e l’area di 375m*2.*

Dati: \(2 p = 80 ( m )\) , \(A = 375 ( m ^{2 } )\)

Obiettivo: \(\overline {AC } \text{ ? }\)

_images/immagini13.png

Soluzione

\(\overline {AC } = \sqrt{\overline {AB } ^{2 } + \overline {BC } ^{2 } }\) per il teorema di Pitagora sul triangolo ABC.

Sono incognite le misura dei lati, quindi poniamo \(\overline {AB } = x \text{ e } \overline {BC } = y \text{ con } x > 0 \text{ e } y > 0\)

Il problema si formalizza con il sistema: \(\{ \begin{array}{l } x + y = 40 \\x \cdot y = 375 \end{array}\) che esprime la ricerca di due numeri nota la loro somma 40 e il loro prodotto 375. I numeri richiesti sono le soluzioni reali positive dell’equazione \(t ^{2 } - 40 t + 375 = 0\) e precisamente \(t _{1 } = 15 \vee t _{2 } = 25\) .

Per come abbiamo disegnato la figura abbiamo quindi: \(AB = 25m ; BC = 15m\) da cui \(\overline {AC } = \sqrt{\overline {AB } ^{2 } + \overline {BC } ^{2 } } = \sqrt{850 } = 5 \sqrt{34 }\) .

Scomposizione del trinomio di secondo grado

Si consideri il trinomio di secondo grado: \(a x ^{2 } + b x + c\) e sia \(a x ^{2 } + b x + c = 0\) (con \(\Delta \geq 0\) ) l’equazione associata a tale trinomio. Effettuiamo le seguenti operazioni:

  • Si mette in evidenza \(a\) : \(a x ^{2 } + b x + c = a \left( x ^{2 } + \frac{b }{a } x + \frac{c }{a } \right)\) ;
  • Si sostituiscono le relazioni trovate nel precedente paragrafo riguardo la somma e il prodotto delle soluzioni \(x _{1 }\) e \(x _{2 }\) : \(a \left( x ^{2 } + \frac{b }{a } x + \frac{c }{a } \right) = a \left[ x ^{2 } - ( x _{1 } + x _{2 } ) x + x _{1 } \cdot x _{2 } \right]\) .
  • Si effettua il raccoglimento parziale: \(a \left[ x ^{2 } - ( x _{1 } + x _{2 } ) x + x _{1 } \cdot x _{2 } \right] = a [ x ^{2 } - x _{1 } x + x _{2 } x + x _{1 } \cdot x _{2 } ]\) e in conclusione: \(a [ x ^{2 } - x _{1 } x + x _{2 } x + x _{1 } \cdot x _{2 } ] = a [ x ( x - x _{1 } ) - x _{2 } ( x - x _{1 } ) ] = a ( x - x _{1 } ) ( x - x _{2 } )\)

È quindi possibile distinguere i casi:

  • I caso : \(\Delta > 0\) Il trinomio di secondo grado può essere scomposto nella forma: \(a ( x - x _{1 } ) ( x - x _{2 } )\) ;
  • II caso: \(\Delta = 0\) Il trinomio di secondo grado può essere scomposto nella forma: \(a ( x - x _{1 } ) ^{2 }\) ;
  • II I caso: \(\Delta < 0\) Il trinomio di secondo grado non può essere scomposto.
Tabella13
Discriminante Scomposizione
\(\Delta > 0 \rightarrow x _{1 } \neq x _{2 }\) \(a x ^{2 } + b x + c = a ( x - x _{1 } ) ( x - x _{2 } )\)
\(\Delta = 0 \rightarrow x _{1 } = x _{2 }\) \(a x ^{2 } + b x + c = a ( x - x _{1 } ) ^{2 }\)
\(\Delta < 0 \rightarrow x _{1, } x _{2 } \notin \mathbb{R}\) \(a x ^{2 } + b x + c\) è irriducibile
_images/immagini11.png

Esempi

  • Scomporre in fattori \(x ^{2 } - 5 x + 6\) . Calcolo le soluzioni dell’equazione \(x ^{2 } - 5 x + 6 = 0\) , \(x _{1,2 } = \frac{5 \pm \sqrt{25 - 24 } }{2 }\) cioè \(x _{1 } = - 2 \vee x _{2 } = 3\) . Applicando la formula ottenuta nel I caso si ha: \(x ^{2 } - 5 x + 6 = ( x - 2 ) ( x + 3 )\)
  • Scomporre in fattori \(x ^{2 } - 12 x + 36\) . Poiché \(\Delta = 144 - 144 = 0\) a pplicando la formula ottenuta nel II caso si ha: \(x ^{2 } - 12 x + 36 = ( x - 6 ) ^{2 }\) , infatti il trinomio è il quadrato di un binomio.
  • Scomporre in fattori \(2 x ^{2 } + 3 x + 5\) . Essendo \(\Delta = 9 - 40 = - 31\) , il trinomio è irriducibile.
  • Scomporre * il trinomio* \(- 5 x ^{2 } + 2 x + 1\) . C alcolo le radici dell’equazione associata \(- 5 x ^{2 } + 2 x + 1 = 0\) : \(x _{1,2 } = \frac{- 2 \pm \sqrt{24 } }{- 10 } = \frac{1 \pm \sqrt{6 } }{5 } \text{ quindi } x _{1 } = \frac{1 - \sqrt{6 } }{5 } \vee x _{2 } = \frac{1 + \sqrt{6 } }{5 }\) e scrivo la scomposizione: \(- 5 x ^{2 } + 2 x + 1 = - 5 \left( x - \frac{1 - \sqrt{6 } }{5 } \right) \left( x - \frac{1 + \sqrt{6 } }{5 } \right)\) .
_images/immagini11.png

Attenzione

  1. Si vuole scomporre in fattori il trinomio \(p = 4 x ^{2 } + 2 x - 6\) , avente tutti i coefficienti pari. Anche se osserviamo che tutti i suoi coefficienti sono pari, NON POSSIAMO DIVIDERE PER DUE, non essendo una equazione; il polinomio \(p = 2 x ^{2 } + x - 3\) è diverso da quello assegnato, mentre le equazioni associate all’uno e all’altro sono equivalenti. Nel procedere alla scomposizione possiamo usare l’equazione \(2 x ^{2 } + x - 3 = 0\) le cui radici sono: \(x _{1 } = - \frac{3 }{2 } \vee x _{2 } = 1\) , e procedere alla scomposizione del trinomio assegnato: \(p = 4 x ^{2 } + 2 x - 6 = 4 \left( x + \frac{3 }{2 } \right) ( x - 1 )\)

Regola di Cartesio

Se in un’equazione di secondo grado i coefficienti sono tutti diversi da zero e il discriminante è non negativo, è possibile avere delle informazioni sui segni delle soluzioni senza calcolarle esplicitamente.

In un’equazione \(a x ^{2 } + b x + c = 0\) , dove i coefficienti sono tutti non nulli, le coppie di coefficienti (a, b) e (b, c) sono dette coppie di coefficienti consecutivi.

Una coppia di coefficienti consecutivi presenta:

;;tab;;una permanenza se i coefficienti hanno lo stesso segno;

;;tab;;una variazione se i coefficienti hanno segni diversi.

Esempi

  \(a\) \(b\) \(c\)
\(+ 2 x ^{2 } - 3 x - 1\) \(+\) \(-\) \(-\)
  variazione permanenza
\(- x ^{2 } - 3 x - 1\) \(-\) \(-\) \(-\)
  permanenza permanenza
\(- 3 x ^{2 } + 4 x - 1\) \(-\) \(+\) \(-\)
  variazione variazione
\(+ 2 x ^{2 } + x - 1\) \(+\) \(+\) \(-\)
  permanenza variazione
TEOREMA DI CARTESIO
In un’equazione di secondo grado \(a x ^{2 } + b x + c = 0\) con \(a , b , c \neq 0\) e \(\Delta \geq 0\) , il numero di radici positive è uguale al numero di variazioni presenti nelle coppie di coefficienti consecutivi. Se vi è una sola variazione, le radici sono discordi e il valore assoluto maggiore è quello della radice positiva se la variazione è nella coppia (a,b), mentre è della radice negativa se la variazione è nella coppia (b,c).

Cerchiamo di capire, attraverso degli esempi, perché i segni dei coefficienti dell’equazione di secondo grado completa hanno una stretta relazione con i segni delle sue soluzioni reali.

_images/immagini11.png

Esempi

  • Dimostrare la seguente affermazione: “ L’equazione * :math:`x ^{2 } + 2 x - 3 = 0` *ha soluzioni reali in quanto \(\Delta = 16 > 0\) ; dal momento che vi è una sola variazione, quello della coppia (b,c), l’equazione ha radici discordi e il valore assoluto maggiore è quello della radice negativa.”

    Dimostriamo quanto è stato affermato tenendo presente che \(x _{1 } + x _{2 } = - \frac{b }{a } \text{ None} \wedge x _{1 } \cdot x _{2 } = \frac{c }{a }\) ; nell’equazione proposta si ha: \(x _{1 } + x _{2 } = - 2 \wedge x _{1 } \cdot x _{2 } = - 3\) dunque prodotto negativo e somma negativa. Il prodotto di due numeri è negativo quando i fattori sono discordi, quindi una soluzione è positiva e una è negativa. Chiamiamo x 1 la soluzione negativa e x 2 la soluzione positiva, poiché \(x _{1 } + x _{2 } = - 2 < 0\) deduciamo che in valore assoluto è più grande il numero negativo, cioè \(\left\lvert x _{1 } \right\rvert > \left\lvert x _{2 } \right\rvert\) .

  • L’equazione \(2 x ^{2 } - 6 x - 56\) ha soluzioni reali in quanto \(\Delta = 484 > 0\) ; dal momento che vi è una sola variazione, l’equazione ha radici discordi e il valore assoluto maggiore è quello della radice positiva dal momento che la variazione è nella coppia (a,b).

  • L’equazione \(- 3 x ^{2 } - 24 x - 21 = 0\) ha soluzioni reali in quanto \(\Delta = 324 > 0\) ; dal momento che non vi sono variazioni, l’equazione ha due radici negative.

  • L’equazione \(x ^{2 } - 10 x + 25 = 0\) ha due soluzioni coincidenti in quanto \(\Delta = 0\) ; dal momento che vi sono due variazioni, le due radici coincidenti sono positive.

_images/immagini11.png

Equazioni parametriche

Equazione parametrica
Si definisce parametrica un’equazione i cui coefficienti dipendono da un parametro.

L’equazione \(3 x ^{2 } + ( k - 1 ) x + ( 2 - 3 k ) = 0\) è parametrica di secondo grado nell’incognita x; i suoi coefficienti dipendono dal valore de l parametro k e quindi la natura e il segno delle sue soluzioni dipendono da k.

In molti problemi di applicazione della matematica in situazioni reali in cui compare un parametro, non interessa tanto determinare le soluzioni dell’equazione che formalizza il problema, quanto sapere se le soluzioni hanno determinate caratteristiche.

Sappiamo che attraverso i coefficienti di un’equazione di secondo grado si possono determinare alcune relazioni tra le sue soluzioni:

  • si hanno soluzioni reali se \(\Delta = b ^{2 } - 4 a c \geq 0\) ; reali coincidenti se \(\Delta = 0\) , reali distinte se \(\Delta > 0\)
  • la somma delle soluzioni è \(x _{1 } + x _{2 } = - \frac{b }{a }\)
  • il prodotto delle soluzioni è \(x _{1 } \cdot x _{2 } = \frac{c }{a }\) .

Nell’equazione \(3 x ^{2 } + ( k - 1 ) x + ( 2 - 3 k ) = 0\) si ha \(\Delta = ( k - 1 ) ^{2 } - 12 ( 2 - 3 k )\) dipendente dal parametro \(k\) .

Dall’analisi del \(\Delta\) si potranno dedurre quali condizioni deve verificare k affinché esistano soluzioni reali. Dall’analisi di somma e prodotto \(x _{1 } + x _{2 } = - \frac{( k - 1 ) }{3 } ; x _{1 } \cdot x _{2 } = \frac{( 2 - 3 k ) }{3 }\) potremo stabilire il segno delle soluzioni reali.

_images/immagini11.png

Esempio

  • Assegnata l’equazione \(( k + 1 ) x ^{2 } + ( 2 k + 3 ) x + k = 0\) stabilire per quale valore di \(k\) : a) L’equazione si riduce al primo grado; b) L’equazione ammette soluzioni reali; distinguere i casi “soluzioni coincidenti” e “soluzioni distinte”, c) La somma delle soluzioni sia nulla; determina in tal caso le soluzioni.

    Svolgimento guidato

    1. l’equazione diventa di primo grado se il coefficiente a si annulla cioè \(k + 1 = 0 \rightarrow k = - 1\) .
    2. studiamo il segno del discriminante: \(\Delta = ( 2 k + 3 ) ^{2 } - 4 k ( k + 1 ) \geq 0\) da cui ricaviamo \(4 k ^{2 } + 12 k + 9 - 4 k ^{2 } - 4 k \geq 0 \rightarrow 8 k + 9 \geq 0\) .
    3. se \(k = - \frac{9 }{8 }\) le soluzioni sono coincidenti;
    4. se \(k > - \frac{9 }{8 }\) le soluzioni sono reali distinte .
    5. dalla formula ricaviamo \(x _{1 } + x _{2 } = - \frac{( 2 k + 3 ) }{( k + 1 ) }\) e quindi ponendo \(2 k + 3 = 0\) si ha somma nulla se \(k = - \frac{3 }{2 }\) . Dopo aver verificato che \(k > - \frac{9 }{8 }\) operando la sostituzione \(k = - \frac{3 }{2 }\) l’equazione diventa \(x ^{2 } - 1 = 0\) con soluzioni \(x _{1 } = - 1 \vee x _{2 } = 1\) .
_images/immagini11.png

Problemi di secondo grado in una incognita

La risoluzione dei problemi … serve ad acuire l’ingegno e a dargli la facoltà di penetrare l’intera ragione di tutte le cose. (R. Descartes)

Sappiamo che nel corso degli studi o nell’attività lavorativa possono presentarsi problemi di diversa natura: di tipo economico, scientifico, sociale; possono riguardare insiemi numerici o figure geometriche. La matematica ci può aiutare a risolvere i problemi quando essi possono essere tradotti in “forma matematica”, quando cioè è possibile trascrivere in simboli le relazioni che intercorrono tra le grandezze presenti nel problema e quando si può costruire, tramite queste relazioni, un modello matematico che ci permetta di raggiungere la soluzione al quesito.

Affronteremo problemi di tipo algebrico o geometrico, che potranno essere formalizzati attraverso equazioni di secondo grado in una sola incognita.

Teniamo presente, prima di buttarci nella risoluzione del problema, alcuni passi che ci aiuteranno a costruire il modello matematico:

  • la lettura “attenta” del testo al fine di individuare l’ambiente del problema, le parole chiave, i dati e le informazioni implicite, l’obiettivo;
  • la scelta della grandezza incognita del problema, la descrizione dell’insieme in cui si ricerca il suo valore, le condizioni che devono essere soddisfatte dall’incognita;
  • la traduzione in “forma matematica” delle relazioni che intercorrono tra i dati e l’obiettivo, cioè l’individuazione del modello matematico (equazione risolvente).

Dopo aver risolto l’equazione occorre confrontare la soluzione trovata con le condizioni poste dal problema.

Problema 1

Nel triangolo rettangolo ABC, rettangolo in C l’ipotenusa supera il cateto maggiore CB di 2m; la differenza tra i cateti è 23m. Determinare la misura del perimetro e l’area di ABC.

_images/immagini38.png

Dati

\(\begin{array}{l } \overline {AB } = \overline {CB } + 2 \\\overline {CB } - \overline {AC } = 23 \\A \widehat {C } B = retto \end{array}\)

Obiettivo

\(\begin{array}{l } \text{ ?} 2 p \\\text{ ?} Area \end{array}\)

Strategia risolutiva.

Osserva che \(2 p = \overline {AB } + \overline {BC } + \overline {AC } ; Area = \frac{\overline {BC } \cdot \overline {AC } }{2 }\)

Poni \(\overline {BC } = x\) dai dati si ha \(\begin{array}{l } \overline {AB } = x + 2 \\\overline {AC } = x - 23 \end{array}\) con \(\{ \begin{array}{l } x > 0 \text{ essendo misura di un segmento} \\x > 23 \text{ poiché } \overline {AC } \text{ deve essere positiva} \end{array}\)

Essendo il triangolo rettangolo, i lati sono legati dal teorema di Pitagora quindi si deve verificare: \(\overline {AB } ^{2 } = \overline {AC } ^{2 } + \overline {BC } ^{2 } \rightarrow ( x + 2 ) ^{2 } = ( x - 23 ) ^{2 } + x ^{2 }\) . Sviluppando i calcoli si ottiene l’equazione risolvente di secondo grado, in forma canonica: è \(x ^{2 } - 50 x + 525 = 0 \text{ con } \Delta = 400\) . L’equazione è determinata con il discriminante positivo, quindi esistono due soluzioni reali distinte: \(x _{1 } = 15 \vee x _{2 } = 35\) entrambe positive. Ai fini del problema \(x _{1 } = 15\) non è accettabile, quindi il problema ha una sola soluzione e \(\overline {BC } = 35 ; \overline {AB } = 37 ; \overline {AC } = 12\)

*Conclusione: * \(2 p = 35 + 37 + 12 = 84 ( m ) ; Area = 210 ( m ^{2 } )\)

Problema 2

*Un padre aveva 26 anni alla nascita del figlio; moltiplicando le età attuali del padre e del figlio si trova il triplo del quadrato dell’età del figlio; calcolare le due età. *

Indichiamo con \(p\) l’età attuale del padre e con \(f\) l’età del figlio

Dati: \(p = f + 26 ; p \cdot f = 3 f ^{2 }\) ;;tab;;Obiettivo: \(\text{ ?} f\) ; \(\text{ ?} p\)

Strategia risolutiva: I dati permettono di impostare la relazione \(( f + 26 ) \cdot f = 3 \cdot f ^{2 }\) che esprime il legame tra le età di oggi del padre e del figlio; siamo di fronte ad un’equazione di secondo grado nell’incognita \(f\) . La soluzione dell’equazione deve essere espressa da un numero positivo poiché esprime l’età.

Risolviamo: \(2 f ^{2 } - 26 f = 0\) le cui soluzioni sono \(f _{1 } = 0 \vee f _{2 } = 13\) . Per le condizioni poste la soluzione del problema è \(f = 13\) .

Risposta: Oggi il figlio ha 13 anni e il padre 39 anni.

Problema 3

Il trapezio isoscele ABCD è inscritto in una semicirconferenza di diametro AB di misura 25cm; determina le * *misure dei lati del trapezio sapendo che il perimetro è 62cm.

_images/immagini5.png

Dati

\(\begin{array}{l } \overline {AB } = 25 \\2 p = 62 \\AB \parallel DC \\\overline {AD } = \overline {CB } \end{array}\)

Obiettivo

\(\begin{array}{l } \text{ ?} \overline {DC } \\\text{ ?} \overline {CB } \end{array}\)

Strategia risolutiva:

\(\overline {AB } + \overline {DC } + 2 \overline {BC } = 62\) ; fissiamo come incognita la misura in cm di BC: \(\overline {BC } = x\)

Determiniamo le condizioni sull’incognita: dovrà essere \(x > 0\) poiché rappresenta la misura di un segmento e inoltre affinché esista realmente il trapezio isoscele il punto C non deve coincidere con il punto medio E dell’arco DC, quindi \(x < \frac{25 }{2 } \sqrt{2 }\)

Tracciata l’altezza \(CH ( H \in AB )\) si ha \(\overline {DC } = \overline {AB } - 2 \overline {HB }\) e per il 1° teorema di Euclide sul triangolo \(ACB\) , rettangolo in \(C\) , \(\overline {HB } \div \overline {CB } = \overline {CB } \div \overline {AB }\) ; determiniamo quindi la misura di \(HB\) in funzione dell’incognita fissata: \(\overline {HB } = \frac{x ^{2 } }{25 } \text{ da cui } \overline {DC } = 25 - \frac{2 x ^{2 } }{25 }\)

Costruiamo l’equazione risolvente: \(25 + 2 x + 25 - \frac{2 x ^{2 } }{25 } = 62 \rightarrow x ^{2 } - 25 x + 150 = 0\) che ha soluzioni \(x _{1 } = 10 \vee x _{2 } = 15\) , entrambe accettabili. Si hanno dunque due trapezi inscritti che risolvono il problema.

_images/immagini8.png

Problema 4

Un capitale di 25000 € viene depositato in banca a un tasso di interesse annuo c. Gli interessi maturati durante il primo anno non vengono ritirati. Nell’anno seguente si investono sia il capitale sia gli interessi maturati a un tasso di interesse annuo aumentato dello 0,5%. Alla fine dei due anni si ritira la somma di 26291,10 €. Calcola i tassi di interesse praticati dalla banca.

Svolgimento. Assumiamo come incognita \(c\) il tasso di interesse praticato il primo anno, espresso come numero decimale e non in forma percentuale. Il tasso praticato nel secondo anno sarà c+0,05.

Alla fine del primo anno in banca rimane tra capitale e interessi \(25000 + 25000 \cdot c = 25000 ( 1 + c )\) . Nel secondo anno il tasso praticato è c+0,005 che va applicato alla somma 25000(1+c).

Si ottiene quindi l’equazione \(25000 ( 1 + c ) ( 1 + c + 0,005 ) = 26291,10\)

Risolviamo l’equazione

\(25000 ( 1 + c ) ( 1,005 + c ) = 26291,10\) moltiplicando tra le parentesi tonde si ha

\(25000 ( 1,005 + c + 1,005 c + c ^{2 } ) = 26291,10\) dividendo per 25000 primo e secondo membro

\(1,005 + c + 1,005 c + c ^{2 } = \frac{26291,10 }{25000 }\) riscrivendo in ordine l’equazione si ha

\(c ^{2 } + 2,005 c - 0,046644\) applicando la formula risolutiva

\(c _{1,2 } = \frac{- 2,005 \pm \sqrt{4,020025 + 0,186576 } }{2 } = \frac{- 2,005 \pm 2,051 }{2 }\)

\(c _{1 } = - 2,028\)

\(c _{2 } = 0,023\)

La soluzione c1 è negativa e non è accettabile.

La risposta al problema è 0,023 cioè 2,3% il primo anno e 2,8% il secondo anno.

Problemi con un parametro

I problemi che abbiamo proposto sono caratterizzati da dati numerici e di conseguenza le soluzioni numeriche dell’equazione risolvente sono facilmente confrontabili con le condizioni poste sull’incognita. Abbiamo anche visto che le soluzioni dell’equazione non sempre sono anche soluzioni del problema e d’altro canto può succedere che il problema abbia due soluzioni.

Affrontiamo ora un problema letterale, nel quale alcuni dati sono espressi da

lettere. In questi problemi dovremo rispettare le condizioni poste sull’incognita, ma anche analizzare per quali valori della lettera il problema ammette soluzioni reali. Dovremo quindi procedere con la discussione dell’equazione parametrica risolvente per stabilire se il problema letterale ammette soluzioni.

Problema 1

Sul lato * :math:`a` * dell’angolo * :math:`a widehat {V } b text{ di 60°}` *si fissano i punti A e B tali che * :math:`overline {VA } = 2 k text{ e } overline {VB } = 8 k` *.

Determina sul lato * :math:`b` * un punto * :math:`P` * in modo che il rapporto tra * :math:`PB` * e * :math:`PA` * sia 2.

_images/immagini37.png

Dati

\(\begin{array}{l } a \hat {V } b = \text{ 60°} \\\overline {VA } = 2 k \\\overline {VB } = 8 k \end{array}\)

Obiettivo

\(\text{ ?} P \in b \text{ tale che } \frac{PB }{PA } = 2\)

Osservazione preliminare : le misure dei segmenti VA e VB sono espresse in forma letterale, affinché il problema abbia significato deve essere \(k > 0\) .

Strategia risolutiva:

La posizione del punto P sul lato b sarà individuata dalla distanza di P da V: poniamo quindi \(\overline {VP } = x \text{ con } x > 0\) e determiniamo \(\overline {PB } \text{ e } \overline {PA }\) in funzione di x per poter sfruttare la richiesta contenuta nell’obiettivo come equazione risolvente.

Sia M il piede della perpendicolare da B al lato b; nel triangolo rettangolo PMB si ha \(\overline {PB } ^{2 } = \overline {BM } ^{2 } + \overline {PM } ^{2 }\) (*) per il teorema di Pitagora. Nel triangolo BVM, rettangolo in M con l’angolo V di 60° si ha \(\overline {BM } = \frac{1 }{2 } \overline {BV } \cdot \sqrt{3 } = 4 k \cdot \sqrt{3 } ; \overline {PM } = \overline {VP } - \overline {VM } \text{ e } \overline {VM } = \frac{1 }{2 } \overline {VB } = 4 k\) ; per quanto detto sul triangolo BVM, quindi \(\overline {PM } = x - 4 k\) ; sostituendo in (*) si ottiene \(\overline {PB } ^{2 } = 48 k ^{2 } + ( x - 4 k ) ^{2 }\) .

Sia N il piede della perpendicolare da A al lato b; nel triangolo rettangolo PNA. Con analogo ragionamento otteniamo: \(\overline {PA } ^{2 } = \overline {AN } ^{2 } + \overline {PN } ^{2 }\) (**) per il teorema di Pitagora. Nel triangolo AVN, rettangolo in N con l’angolo V di 60° si ha \(\overline {AN } = \frac{1 }{2 } \overline {AV } \cdot \sqrt{3 } = k \cdot \sqrt{3 } \text{ e } \overline {VN } = \frac{1 }{2 } \overline {AV } = k ; \overline {PN } = \overline {VP } - \overline {VN } = x - k\) ; sostituendo in (**) si ottiene \(\overline {PA } ^{2 } = 3 k ^{2 } + ( x - k ) ^{2 }\) .

Determiniamo l’equazione risolvente ricordando che il rapporto tra due segmenti è uguale al rapporto tra le rispettive misure ed elevando al quadrato si ha \(\frac{\overline {PB } ^{2 } }{\overline {PA } ^{2 } } = 4\) . Sostituendo quanto trovato si ha l’equazione \(48 k ^{2 } + ( x - 4 k ) ^{2 } = 4 \cdot \left[ 3 k ^{2 } + ( x - k ) ^{2 } \right] \text{ da cui } x ^{2 } = 16 k ^{2 }\) . Si tratta di un’equazione di secondo grado pura, avente due soluzioni reali opposte essendo il secondo membro positivo, quindi \(x _{1 } = - 4 k \vee + 4 k\) e per le condizioni poste solo x2 è accettabile.

Con quale punto della figura tracciata inizialmente viene a coincidere il punto P che risolve il problema?

Eesercizi

Risoluzione di una equazione di secondo grado pura

  1. \(x ^{2 } - 1 = 0\) ; \(x ^{2 } = \frac{49 }{25 }\) ; \(2 x ^{2 } - 32 = 0\) ;;tab;;R. \(x _{1 } = + 4 \vee x _{2 } = - 4\)
  2. \(x ^{2 } - 25 = 0\) ; \(16 x ^{2 } = 1\) ; \(3 x ^{2 } + 3 = 0\) ;;tab;;R. \(I.S. = \varnothing\)
  3. \(x ^{2 } - 9 = 0\) ; \(25 = 9 x ^{2 }\) ; \(x ^{2 } - 3 = 0\) ;;tab;;R. \(x _{1 } = \sqrt{3 } \vee x _{2 } = - \sqrt{3 }\)
  4. \(x ^{2 } + 36 = 0\) ; \(4 - x ^{2 } = 0\) ; \(x ^{2 } + 4 = 0\) ;;tab;;R. \(I.S. = \varnothing\)
  5. \(x ^{2 } = 49\) ; \(4 - 9 x ^{2 } = 0\) ; \(5 x ^{2 } - 3 = 0\) ;;tab;;R. \(x _{1 } = \frac{\pm \sqrt{15 } }{5 }\)
  6. \(4 x ^{2 } - 9 = 0\) ; \(9 x ^{2 } - 25 = 0\) ; \(6 x ^{2 } = 0\) ;;tab;;R. \(x _{1,2 } = 0\)
  7. \(2 x ^{2 } - 1 = 0\) ; \(4 x ^{2 } + 16 = 0\) ; \(1 + x ^{2 } = 50\) ;;tab;;R. \(x _{1,2 } = \pm 7\)
  8. \(3 x ^{2 } - 1 = 0\) ; \(27 x ^{2 } - 3 = 0\) ; \(7 x ^{2 } = 28\) ;;tab;;R. \(x _{1,2 } = \pm 2\)
  9. \(4 x ^{2 } - 4 = 0\) ; \(5 x ^{2 } - 125 = 0\) ; \(0,04 x ^{2 } = 1\) ;;tab;;R. \(x _{1,2 } = \pm 5\)
  10. \(x ^{2 } - 0,01 = 0\) ; \(0,5 x ^{2 } - 4,5 = 0\) ; \(0,09 x ^{2 } = 0,01\) ;;tab;;R. \(x _{1,2 } = \pm \frac{1 }{3 }\)
  11. \(\frac{1 }{2 } x ^{2 } - 2 = 0\) ; \(x ^{2 } – \frac{9 }{4 } = 0\) ; \(x ^{2 } – \frac{1 }{6 } = 0\) ;;tab;;R. \(x _{1,2 } = \pm \frac{\sqrt{6 } }{6 }\)
  12. \(121 x ^{2 } - \frac{1 }{169 } = 0\) ; \(x ^{2 } + \frac{9 }{4 } = 0\) ; \(4 \left( x ^{2 } - \frac{3 }{4 } \right) = 13\) ;;tab;;R. \(x _{1,2 } = \pm 2\)
  13. \(x ^{2 } - \sqrt{3 } = 0\) ; \(- 9 x ^{2 } = - 1\) ; \(4 x ^{2 } = - 9\) ;;tab;;R. \(I.S. = \varnothing\)
  14. \(x ^{2 } + 6 = 42\) ; \(5 - 125 x ^{2 } = 0\) ; \(18 - x ^{2 } = 0\) ;;tab;;R. \(x _{1,2 } = \pm 3 \sqrt{2 }\)
  15. \(( x + 3 ) ^{2 } = 6 x + 34\) ; \(( x + 1 ) ^{2 } = 25\) ; \(( x - \sqrt{3 } ) ( x + \sqrt{3 } ) = 13\) ;;tab;;R. \(x _{1,2 } = \pm \sqrt{10 }\)
  16. \(( x + \sqrt{2 } ) ^{2 } = 2 \sqrt{2 } x\) ; \(( x - 2 ) ^{2 } + ( 1 - x ) ^{2 } = 1 - 6x\) ; \(( \sqrt{2 } x - \sqrt{3 } ) ( \sqrt{2 } x + \sqrt{3 } ) = 0\) ; ;;tab;; R. \(x _{1,2 } = \pm \frac{\sqrt{6 } }{2 }\)

Risoluzione di un’equazione incompleta spuria

  1. \(x ^{2 } - 3 x = 0\) ; \(3 x ^{2 } - 2 x = 0\) ;;tab;;R. \(x _{1 } = 0 \vee x _{2 } = \frac{2 }{3 }\)
  2. \(x ^{2 } - 3 x = 0\) ; \(7 x ^{2 } + 2 x = 0\) ;;tab;;R. \(x _{1 } = 0 \vee x _{2 } = - \frac{2 }{7 }\)
  3. \(x ^{2 } + 2 x = 0\) ; \(x ^{2 } + 5 x = 0\) ;;tab;;R. \(x _{1 } = 0 \vee x _{2 } = - 5\)
  4. \(x ^{2 } - x = 0\) ; \(18 x ^{2 } - 36 x = 0\) ;;tab;;R. \(x _{1 } = 0 \vee x _{2 } = 2\)
  5. \(2x ^{2 } + 6x = 0\) ; \(1000 x - 2000 x ^{2 } = 0\) ;;tab;;R. \(x _{1 } = 0 \vee x _{2 } = \frac{1 }{2 }\)
  6. \(9x ^{2 } + 16x = 0\) ; \(6 x ^{2 } = 5 x\) ;;tab;;R. \(x _{1 } = 0 \vee x _{2 } = \frac{5 }{6 }\)
  7. \(5x = 25x ^{2 }\) ; \(3 x ^{2 } - 2 x = 4 x\) ;;tab;;R. \(x _{1 } = 0 \vee x _{2 } = 2\)
  8. \(81x ^{2 } = 9x\) ; \(0,1 x ^{2 } - 0,5 x = 0\) ;;tab;;R. \(x _{1 } = 0 \vee x _{2 } = 5\)
  9. \(7x ^{2 } - 2x = 0\) ; \(0,5 x ^{2 } + 0,1 x = 0\) ;;tab;;R. \(x _{1 } = 0 \vee x _{2 } = - 0,2\)
  10. \(x ^{2 } + \frac{1 }{2 } x = 0\) ; \(\frac{1 }{2 } x - \frac{1 }{4 } x ^{2 } = 0\) ;;tab;;R. \(x _{1 } = 0 \vee x _{2 } = 2\)
  11. \(\sqrt{2 } x ^{2 } + \sqrt{3 } x = 0\) ; \(x ^{2 } + \sqrt{2 } x = 0\) ;;tab;;R. \(x _{1 } = 0 \vee x _{2 } = - \sqrt{2 }\)
  12. \(- 2x ^{2 } + 4x = 0\) ; \(5 \sqrt{2 } x ^{2 } - 2 \sqrt{2 } x = 0\) ;;tab;;R. \(x _{1 } = 0 \vee x _{2 } = \frac{2 }{5 }\)
  13. \(\frac{1 }{6 } x ^{2 } + \frac{1 }{4 } x = 0\) ; \(3x ^{2 } - \frac{4 }{3 } x = 0\) ;;tab;;R. \(x _{1 } = 0 \vee x _{2 } = \frac{4 }{9 }\)
  14. \(( x - 2 ) ^{2 } = 4\) ; \(( x + 1 ) ^{2 } = 1\) ;;tab;;R. \(x _{1 } = 0 \vee x _{2 } = - 2\)
  15. \(( x + \sqrt{2 } ) ^{2 } = 2\) ; \(77 x - 11 x ^{2 } = 0\) ;;tab;;R. \(x _{1 } = 0 \vee x _{2 } = 7\)
  16. \(\frac{3 }{4 } x ^{2 } - \frac{3 }{2 } x = 0\) ; \(\frac{11 }{3 } x ^{2 } = - 2 x\) ;;tab;;R. \(x _{1 } = 0 \vee x _{2 } = - \frac{6 }{11 }\)
  17. \(\frac{1 }{2 } ( x - 2 ) ^{2 } - x = 2\) ;;tab;;R. \(x _{1 } = 0 \vee x _{2 } = 6\)
  18. \(( x - 1 ) ( x + 3 ) = 3 x ^{2 } - 3\) ;;tab;;R. \(x _{1 } = 0 \vee x _{2 } = 1\)
  19. \(( 3 x - 2 ) ^{2 } - 4 = 6 x ^{2 }\) ;;tab;;R. \(x _{1 } = 0 \vee x _{2 } = 4\)
  20. \(( x - 2 ) ^{2 } + ( 1 - x ) ^{2 } = 5\) ;;tab;;
  21. \(( x - 2 ) ^{3 } - 4 ( 2 x - 1 ) = ( x + 2 ) \left( x ^{2 } - 2 x + 4 \right) - 12\)
  22. \(( \sqrt{2 } + x ) ^{3 } - ( \sqrt{3 } + x ) ^{3 } = 2 \sqrt{2 } - 3 \sqrt{3 }\) ;;tab;;R. \(x _{1 } = 0 \vee x _{2 } = - ( \sqrt{2 } + \sqrt{5 } )\)
  23. \(( \sqrt{2 } x - \sqrt{3 } ) ( \sqrt{2 } x + \sqrt{3 } ) + ( \sqrt{3 } x + \sqrt{3 } ) ^{2 } + ( x - 1 ) ^{2 } = 1\)
  24. \(\left( x ^{2 } + \sqrt{2 } \right) ( \sqrt{3 } - 1 ) + ( 2 x + \sqrt{3 } ) ( \sqrt{2 } - 1 ) - \sqrt{2 } + \sqrt{3 } = 0\)

Risoluzione di un’equazione completa

Risolvi le seguenti equazioni complete

  1. \(x ^{2 } - 5 x + 6 = 0\) R. \(x _{1 } = 2 \vee x _{2 } = 3\) ; \(x ^{2 } + x - 20 = 0\) ;;tab;;R. \(x _{1 } = - 5 \vee x _{2 } = 4\)
  2. \(2 x ^{2 } - 6 x - 6 = 0\) R. \(x _{1,2 } = \frac{3 \pm \sqrt{21 } }{7 }\) ; \(x ^{2 } - 3 x + 6 = 0\) ;;tab;;R. \(I.S. = \varnothing\)
  3. \(- x ^{2 } + x + 42 = 0\) R. \(x _{1 } = - 6 \vee x _{2 } = 7\) ; \(- x ^{2 } + 10 x - 25 = 0\) ;;tab;;R. \(x _{1 } = x _{2 } = 5\)
  4. \(- 2 x ^{2 } + 7 x - 5 = 0\) R. \(x _{1 } = 1 \vee x _{2 } = \frac{5 }{2 }\) ; \(3 x ^{2 } + 2 x - 1 = 0\) ;;tab;;R. \(x _{1 } = - 1 \vee x _{2 } = \frac{1 }{3 }\)
  5. \(2 x ^{2 } - \sqrt{5 } x - 1 = 0\) R. \(x _{1,2 } = \frac{\sqrt{5 } \pm \sqrt{13 } }{4 }\) ; \(x ^{2 } - 2 \sqrt{3 } x - 4 = 0\) ;;tab;;R \(x _{1,2 } = \sqrt{3 } \pm \sqrt{7 }\)
  6. \(x ^{2 } - 3 x - 2 = 0\) R. \(x _{1,2 } = \frac{3 \pm \sqrt{17 } }{2 }\) ; \(2 x ^{2 } - \sqrt{5 } x - 1 = 0\) ;;tab;;R. \(x _{1 } = - \sqrt{2 } \vee x _{2 } = \frac{3 \sqrt{2 } }{2 }\)
  7. \(- \frac{4 }{3 } x ^{2 } - x + \frac{3 }{2 } = 0\) R. \(x _{1 } = - \frac{3 }{2 } \vee x _{2 } = \frac{3 }{4 }\) ; \(- \frac{4 }{5 } x ^{2 } + \frac{1 }{2 } x - \frac{1 }{20 } = 0\) ;;tab;;R. \(x _{1 } = \frac{1 }{8 } \vee x _{2 } = \frac{1 }{2 }\)
  8. \(- x ^{2 } + 4 x - 7 = 0\) R. \(I.S. = \varnothing\) ; \(x ^{2 } - \sqrt{5 } x - \sqrt{5 } = 0\) ;;tab;;R \(x _{1,2 } = \frac{\sqrt{5 } \pm \sqrt{5 + 4 \sqrt{5 } } }{2 }\)
  9. \(x ^{2 } - 5 x + 3 = 0\) R. \(x _{1,2 } = \frac{5 \pm \sqrt{13 } }{2 }\) ; \(x ^{2 } - 4 x + 9 = 0\) ;;tab;;R. \(I.S. = \varnothing\)
  10. \(x ^{2 } - 4 x - 9 = 0\) R. \(x _{1,2 } = 2 \pm \sqrt{13 }\) ; \(x ^{2 } + 6 x - 2 = 0\) ;;tab;;R. \(x _{1,2 } = - 3 \pm \sqrt{11 }\)
  11. \(x ^{2 } - 3 x - \frac{5 }{2 } = 0\) R. \(x _{1,2 } = \frac{3 \pm \sqrt{19 } }{2 }\) ; \(2 x ^{2 } - 3 x + 1 = 0\) ;;tab;;R. \(x _{1 } = 1 \vee x _{2 } = \frac{1 }{2 }\)
  12. \(\frac{4 }{3 } x ^{2 } - \frac{1 }{3 } x - 1 = 0\) R. \(x _{1 } = 1 \vee x _{2 } = - \frac{3 }{4 }\) ; \(3 x ^{2 } + x - 2 = 0\) ;;tab;;R. \(x _{1 } = - 1 \vee x _{2 } = \frac{2 }{3 }\)
  13. \(3 x ^{2 } - \frac{2 }{3 } x - 1 = 0\) R. \(x _{1,2 } = \frac{1 \pm 2 \sqrt{7 } }{9 }\) ; \(\sqrt{2 } x ^{2 } - x - 3 \sqrt{2 } = 0\) ;;tab;;R. \(x _{1 } = - \sqrt{2 } ; x _{2 } = \frac{3 \sqrt{2 } }{2 }\)
  14. \(x ^{2 } - ( \sqrt{2 } + \sqrt{3 } ) x + \sqrt{6 } = 0\) R. \(x _{1 } = \sqrt{2 } \vee x _{2 } = \sqrt{3 }\) ; \(x ^{2 } + ( \sqrt{2 } - \sqrt{3 } ) x - \sqrt{6 } = 0\) ;;tab;;R. \(x _{1 } = - \sqrt{2 } \vee x _{2 } = \sqrt{3 }\)
  15. \(( 3 x + 1 ) ^{2 } - ( 2 x + 2 ) ^{2 } = 0\) R. \(x _{1 } = - \frac{3 }{5 } \vee x _{2 } = 1\) ; \(( x + 5 ) ^{2 } = 5 ( 4 x + 5 )\) ;;tab;;R. \(x _{1 } = 0 \vee x _{2 } = 10\)
  16. \(( x - 2 ) ( 3 - 2 x ) = x - 2\) R. \(x _{1 } = 1 \vee x _{2 } = 2\) ; \(( x + 200 ) ^{2 } + x + 200 = 2\) ;;tab;;R. \(x _{1 } = - 202 \vee x _{2 } = - 199\)
  17. \(( x ^{2 } + x + 1 ) ( x ^{2 } - x - 1 ) = ( x ^{2 } - 1 ) ^{2 }\) ;;tab;;R. \(x _{1,2 } = 1 \pm \sqrt{3 }\)

Risolvi le seguenti equazioni, applicando quando possibile la formula ridotta o ridottissima.

  1. \(3 x ^{2 } - 2 x - 2 = 0\) ;;tab;;R. \(x _{1 } = \frac{1 + \sqrt{7 } }{3 } \vee x _{2 } = \frac{1 - \sqrt{7 } }{3 }\)
  2. \(x ^{2 } + 6 x - 3 = 0\) ;;tab;;R. \(x _{1 } = - 3 + 2 \sqrt{3 } \vee x _{2 } = - 3 - 2 \sqrt{3 }\)
  3. \(4 x ^{2 } - 8 x + 3 = 0\) ;;tab;;R. \(x _{1 } = \frac{1 }{2 } \vee x _{2 } = \frac{3 }{2 }\)
  4. \(7 x ^{2 } - 2 x - 5 = 0\) ;;tab;;R. \(x _{1 } = 1 \vee x _{2 } = - \frac{5 }{7 }\)
  5. \(40 x ^{2 } + 80 x - 30 = 0\) ;;tab;;R. \(x _{1 } = \frac{- 2 + \sqrt{7 } }{2 } \vee x _{2 } = \frac{- 2 - \sqrt{7 } }{2 }\)
  6. \(5 x ^{2 } - 4 x + 1 = 0\) ;;tab;;R. \(I.S. = \varnothing\)
  7. \(5 x ^{2 } - 4 x - 9 = 0\) ;;tab;;R. \(x _{1 } = - 1 \vee x _{2 } = \frac{9 }{5 }\)
  8. \(\frac{3 }{2 } x ^{2 } + 2 x - \frac{3 }{4 } = 0\) ;;tab;;R. \(x _{1 } = \frac{- 4 + \sqrt{34 } }{6 } \vee x _{2 } = - \frac{4 + \sqrt{34 } }{6 }\)
  9. \(6 x ^{2 } - 4 x - 2 = 0\) ;;tab;;R. \(x _{1 } = 1 \vee x _{2 } = - \frac{1 }{3 }\)
  10. \(90 x ^{2 } - 180 x - 270 = 0\) ;;tab;;R. \(x _{1 } = 3 \vee x _{2 } = - 1\)
  11. \(\frac{3 }{2 } x ^{2 } - 4 x + 2 = 0\) ;;tab;;R. \(x _{1 } = 2 \vee x _{2 } = \frac{2 }{3 }\)
  12. \(\frac{4 }{3 } x ^{2 } - 6 x + 6 = 0\) ;;tab;;R. \(x _{1 } = 3 \vee x _{2 } = \frac{3 }{2 }\)
  13. \(x ^{2 } - 6 x + 1 = 0\) ;;tab;;R. \(x _{1 } = 3 + 2 \sqrt{2 } \vee x _{2 } = 3 - 2 \sqrt{2 }\)
  14. \(3 x ^{2 } - 12 x - 3 = 0\) ;;tab;;R. \(x _{1 } = 2 + \sqrt{5 } \vee x _{2 } = 2 - \sqrt{5 }\)
  15. \(7 x ^{2 } - 6 x + 8 = 0\) ;;tab;;R. \(I.S. = \varnothing\)
  16. \(3 x ^{2 } - 18 x + 27 = 0\) ;;tab;;R. \(x _{1 } = x _{2 } = 3\)
  17. \(9 x ^{2 } + 12 x + 1 = 0\) ;;tab;;R. \(x _{1 } = \frac{- 2 + \sqrt{3 } }{3 } \vee x _{2 } = - \frac{2 + \sqrt{3 } }{3 }\)
  18. \(9 x ^{2 } - 12 x + 4 = 0\) ;;tab;;R. \(x _{1 } = x _{2 } = \frac{2 }{3 }\)
  19. \(4 x ^{2 } - 32 x + 16 = 0\) ;;tab;;R. \(x _{1 } = 4 + 2 \sqrt{3 } \vee x _{2 } = 4 - 2 \sqrt{3 }\)
  20. \(3 x ^{2 } + 10 x + 20 = 0\) ;;tab;;R. \(I.S. = \varnothing\)

Esercizi vari sulle equazioni di 2° grado

  1. \(( 3 x + 1 ) \left( \frac{5 }{2 } + x \right) = 2 x - 1\) ;;tab;;R. \(x _{1 } = - 1 \vee x _{2 } = - \frac{7 }{6 }\)

  2. \(( 3 x - 2 ) ^{2 } + ( 5 x - 1 ) ^{2 } = ( 3 x - 2 ) ( 5 x - 1 )\) ;;tab;;R. \(I.S. = \varnothing\)

  3. \(3 x - x ^{2 } = x ^{2 } + 3 ( x - 2 )\) ;;tab;;R. \(x _{1 } = \sqrt{3 } \vee x _{2 } = - \sqrt{3 }\)

  4. \(2 ( x - 1 ) ( x + 1 ) = 2\) ;;tab;;R. \(x _{1 } = - \sqrt{2 } \vee x _{2 } = + \sqrt{2 }\)

  5. \(( 2 x - 1 ) ( 4 - x ) - 11 x = ( 1 - x ) ^{2 }\) ;;tab;;R. \(I.S. = \varnothing\)

  6. \(2x ^{2 } = x + x ^{2 } - ( x + \sqrt{x } ) ( x - \sqrt{x } )\)

  7. \(( x - 3 ) ^{2 } = 9 - 6 x\) ;;tab;;R. \(x _{1 } = x _{2 } = 0\)

  8. \(( x - 2 ) ^{3 } - 1 = x ^{3 } + 12 x - 11\) ;;tab;;R. \(x _{1 } = \frac{\sqrt{3 } }{3 } \vee x _{2 } = - \frac{\sqrt{3 } }{3 }\)

  9. \(\frac{3 x - 2 }{2 } = x ^{2 } - 2\) ;;tab;;R. \(x _{1 } = 2 \vee x _{2 } = - \frac{1 }{2 }\)

  10. \(( 2 x - 3 ) ( 2 x + 3 ) = 27\)

    ;;tab;;R.

    \(x _{1 } = - 3 \vee x _{2 } = + 3\)

  11. \(\frac{x - 3 }{2 } - \frac{x ^{2 } + 2 }{3 } = 1 + x\) ;;tab;;R. \(I.S. = \varnothing\)

  12. \(\frac{x - 2 }{3 } - ( 3 x + 3 ) ^{2 } = x\) ;;tab;;R. \(x _{1 } = - 1 \vee x _{2 } = - \frac{29 }{27 }\)

  13. \(( x - 2 ) ^{3 } - x ^{3 } = x ^{2 } - 4\) ;;tab;;R. \(x _{1,2 } = \frac{6 \pm 2 \sqrt{2 } }{7 }\)

  14. \(x ( 1 - 5 x ) = [ 3 - ( 2 + 5 x ) ] x - ( x ^{2 } - 1 )\) ;;tab;;R. \(x _{1 } = - 1 \vee x _{2 } = + 1\)

  15. \(( x + 1 ) ^{3 } - ( x + 2 ) ^{2 } = \frac{2 x ^{3 } - 1 }{2 }\) ;;tab;;R. \(x _{1,2 } = \frac{1 \pm \sqrt{21 } }{4 }\)

  16. \(\frac{( x - 1 ) ^{2 } }{2 } - \frac{2 x - 5 }{3 } = - \frac{5 }{3 } x\) ;;tab;;R. \(I.S. = \varnothing\)

  17. \(( x + 2 ) ^{3 } + 4 x ^{2 } = ( x - 2 ) ^{3 } + 16\) ;;tab;;R. \(x _{1 } = x _{2 } = 0\)

  18. \(( 2 - x ) ^{3 } - ( 2 - x ) ^{2 } = \frac{3 - 4 x ^{3 } }{4 }\) ;;tab;;R. \(I.S. = \varnothing\)

  19. \(3 \left( x + \sqrt{2 } \right) ^{2 } - 18 \left( x + \sqrt{2 } \right) + 27 = 0\) ;;tab;;R. \(x _{1 } = x _{2 } = 3 - \sqrt{2 }\)

  20. \(( 4 - 3 x ) ^{3 } + 27 x ^{3 } = 64 + 24 x\) ;;tab;;R. \(x _{1 } = 0 \vee x _{2 } = \frac{14 }{9 }\)

  21. \(\left( \frac{x - 1 }{3 } - \frac{x }{6 } \right) ^{2 } = ( x + 1 ) ^{2 }\) ;;tab;;R. \(x _{1 } = - \frac{8 }{5 } \vee x _{2 } = - \frac{4 }{7 }\)

  22. \(( \sqrt{3 } x + 1 ) ^{2 } + ( \sqrt{3 } x - 1 ) ^{2 } - 3 ( \sqrt{3 } x + 1 ) ( \sqrt{3 } x - 1 ) = 0\) ;;tab;; \(x _{1,2 } = \pm \sqrt{\frac{5 }{3 } }\)

  23. \(\frac{( 2 x + 1 ) ( x - 2 ) }{3 } + \frac{( x + \sqrt{5 } ) ( x - \sqrt{5 } ) }{2 } = \frac{( x - 1 ) ^{2 } }{6 }\) ;;tab;; \(x _{1,2 } = \frac{1 \pm \sqrt{31 } }{3 }\)

  24. \(\left( \frac{1 }{2 } x + 1 \right) ^{3 } = \left( \frac{1 }{2 } x - 1 \right) \left( \frac{1 }{2 } x + 1 \right) ^{2 }\) ;;tab;; \(x _{1 } = x _{2 } = - 2\)

  25. \(\frac{( 3 x - 1 ) ^{2 } }{3 } - \frac{( 1 - 2 x ) ^{2 } }{5 } + \frac{3 x ( x - 1 ) }{5 } + \frac{( 1 + x ) ^{2 } }{3 } = 0\) ;;tab;;R. \(I.S. = \varnothing\)

  26. \(\frac{1 }{\sqrt{10 } } x ^{2 } + 1 = \left( \frac{1 }{\sqrt{2 } } + \frac{1 }{\sqrt{5 } } \right) x\)

  27. \(( 3 x - 1 ) ^{2 } + ( 2 x + 1 ) ^{2 } = ( 3 x - 1 ) ( 2 x + 1 )\) ;;tab;;R. \(I.S. = \varnothing\)

  28. \(( x + 1 ) ^{4 } - ( x + 1 ) ^{3 } = x ^{3 } ( x + 4 ) - x ( x + 1 ) ^{2 } + 3 x\) ;;tab;;R. \(x _{1 } = 0 \vee x _{2 } = \frac{1 }{5 }\)

  29. \(\left( \frac{1 }{2 } x ^{2 } + 1 \right) ^{3 } + \frac{1 }{6 } x ^{3 } = \left( \frac{1 }{2 } x ^{2 } - 1 \right) ^{3 } + \frac{1 }{6 } ( x + 1 ) ^{3 } + \frac{3 }{2 } x ^{4 }\) ;;tab;;R. \(x _{1,2 } = \frac{- 3 \pm \sqrt{141 } }{6 }\)

  30. \(\frac{x - 2 }{2 } \cdot \frac{x + 2 }{3 } + \frac{1 }{3 } \left[ \frac{1 }{2 } - \left( x + \frac{1 }{2 } \right) \right] + 4 \left( x - \frac{1 }{2 } \right) \left( x + \frac{1 }{2 } \right) + \frac{5 }{3 } = 0\) ;;tab;;R. \(x _{1 } = 0 \vee x _{2 } = \frac{2 }{25 }\)

  31. \(( 2 - 3 x ) ^{2 } - 1 = 8 ( 1 - 2 x ) + ( 2 x + 1 ) ^{2 } - 1\) ;;tab;;R. \(x _{1 } = - 1 \vee x _{2 } = + 1\)

  32. \(x ^{2 } + \left( \sqrt{3 } - \sqrt{2 } \right) x - \sqrt{6 } = 0\) ;;tab;;R. \(x _{1 } = - \sqrt{3 } \vee x _{2 } = + \sqrt{2 }\)

  33. \(\frac{2 \sqrt{3 } x + 1 }{\sqrt{2 } } - \left( x - \sqrt{3 } \right) ^{2 } = \frac{1 - 3 \sqrt{2 } x }{\sqrt{2 } } + \sqrt{3 } x \left( \sqrt{2 } + 2 \right)\) ;;tab;;R. \(I.S. = \varnothing\)

  34. \(\sqrt{3 } ( 2 x - 30 ) ^{2 } - 2 \sqrt{27 } ( 60 - 4 x ) = 0\) ;;tab;;R. \(x _{1 } = 9 ; x _{2 } = 15\)

  35. \(\left( 2 x + \frac{1 }{2 } \right) ^{2 } - \frac{1 }{2 } \left( \frac{1 }{2 } x - 1 \right) ^{2 } + \left( x - \frac{1 }{2 } \right) \left( x + \frac{1 }{2 } \right) = 0\) ;;tab;;R. \(x _{1 } = - \frac{2 }{3 } \vee x _{2 } = \frac{2 }{13 }\)

  36. \(\frac{x ^{2 } - 16 }{9 } + \frac{( x - 1 ) ^{2 } }{3 } = \frac{x ( x - 2 ) }{9 } + \left( x - \frac{5 }{2 } \right) \left( x + \frac{1 }{3 } \right)\) ;;tab;;R. \(x _{1,2 } = \frac{31 \pm \sqrt{433 } }{24 }\)

  37. \(\frac{( x - 1 ) ( x + 2 ) }{2 } + \frac{( x + 2 ) ( x - 3 ) }{3 } = \frac{( x - 3 ) ( x + 4 ) }{6 }\) ;;tab;;R. \(x _{1 } = \pm \frac{\sqrt{6 } }{2 }\)

  38. \(\left( 2 x - \frac{1 }{2 } \right) ^{2 } + \left( \frac{x - 1 }{2 } - \frac{x }{3 } \right) x = - x ^{2 } + \frac{2 }{3 } \left( x - \frac{1 }{2 } \right) x - \frac{1 }{2 } x + \frac{1 }{9 }\) ;;tab;;R. \(x _{1,2 } = \frac{10 \pm \sqrt{10 } }{54 }\)

  39. \(\frac{1 }{4 } ( 2 x - 1 ) ^{2 } - \frac{1 }{3 } ( x - 1 ) ^{2 } + \frac{( x - 2 ) ( x + 2 ) }{2 } - \frac{1 }{6 } x + \frac{1 }{6 } = 0\) ;;tab;;R. \(x _{1,2 } = \frac{3 \pm \sqrt{331 } }{14 }\)

  40. \(\frac{1 }{2 } ( 2 x - 1 ) ( x + 1 ) + \frac{1 }{3 } \left( x ^{2 } - 5 \right) + 2 x ( x - 1 ) ( x + 1 ) = 2 ( x + 2 ) ^{3 } - ( 2 x - 1 ) ^{2 }\) ;;tab;;R. \(x _{1,2 } = \frac{- 177 \pm \sqrt{14849 } }{80 }\)

  41. \(\frac{3 x - 1 }{\sqrt{5 } - \sqrt{3 } } + \frac{\left( x - \sqrt{3 } \right) \left( x + \sqrt{3 } \right) }{\sqrt{3 } } - \frac{\left( x - \sqrt{3 } \right) ^{2 } }{\sqrt{3 } } = \frac{x ^{2 } }{\sqrt{5 } - \sqrt{3 } } + 2 x - 2 \sqrt{3 }\) ;;tab;;R. \(x _{1,2 } = \frac{3 \pm \sqrt{5 } }{2 }\)

  42. \(\left( x + \frac{1 }{2 } \right) ^{2 } - \frac{3 x ^{2 } - 7 x + 2 }{2 } – \frac{x }{4 } + \frac{5 x - 13 }{2 } = \frac{2 }{3 } x ( 1 - x ) + \frac{73 }{12 } x - \frac{15 }{12 }\) ;;tab;;R. \(x _{1 } = - 6 \vee x _{2 } = + 6\)

  43. \(\frac{( x ^{2 } + 2 x + 1 ) ^{2 } }{4 } + \frac{( x + 1 ) ^{2 } }{2 } + \frac{( x ^{4 } - 1 ) }{8 } - ( 2 x ^{2 } - 2 x + 1 ) ^{2 } + 9 x ^{3 } \left( \frac{3 }{8 } x - 1 \right) + \frac{1 }{4 } x ^{2 } ( x ^{2 } + 20 ) = 0\) R. \(x _{1,2 } = 3 \pm \frac{\sqrt{138 } }{4 }\)

Equazioni che si possono risolvere con opportune sostituzioni

Risolvi le seguenti equazioni con opportune sostituzioni:

  1. \(( 4x + 3 ) ^{2 } = 25\) ;;tab;;R. \(x _{1 } = - 2 \vee x _{2 } = \frac{1 }{2 }\)
  2. \(( x - 5 ) ^{2 } + 9 = 0\) ;;tab;;
  3. \(( 3x - 1 ) ^{2 } - 36 = 0\)
  4. \(4 ( 2x + 1 ) ^{2 } = 36\)
  5. \(( 3x - 5 ) ^{2 } - 49 = 0\) ;;tab;;R. \(x _{1 } = 4 \vee x _{2 } = - \frac{2 }{3 }\)
  6. \(3 ( 2x + 5 ) ^{2 } - 4 ( 2x + 5 ) = 0\) ;;tab;;
  7. \(( 3 \cdot 10 ^{3 } x - 10 ) ^{2 } - 5 ( 3 \cdot 10 ^{3 } x - 10 ) = - 6\) ;;tab;;
  8. \(( x - 1 ) ^{2 } - ( \sqrt{3 } + \sqrt{5 } ) ( x - 1 ) + \sqrt{15 } = 0\) ;;tab;; \(x _{1 } = 1 + \sqrt{3 } \vee x _{2 } = 1 + \sqrt{5 }\)
  9. \(3 ( 1 - 2x ) ^{2 } - 2 ( 1 - 2x ) - 1 = 0\)
  10. \(\frac{4 }{3 } ( x - 2 ) ^{2 } - 6 ( x - 2 ) + 6 = 0\) ;;tab;;R. \(x _{1 } = 5 \vee x _{2 } = \frac{7 }{2 }\)
  11. \(\frac{1 }{2 } \left( x - \frac{1 }{2 } \right) ^{2 } - 2 \left( x - \frac{1 }{2 } \right) = 0\)
  12. \(2 ( x ^{2 } - 1 ) ^{2 } + 3 ( x ^{2 } - 1 ) - 5 = 0\)
  13. \(3 ( 34 x - 47 ) ^{2 } - 2 ( 34 x - 47 ) = 1\) ;;tab;;R. \(x _{1 } = \frac{24 }{17 } \vee x _{2 } = \frac{70 }{51 }\)
  14. \(( x - \sqrt{( 2 ) } ) ^{2 } - ( \sqrt{( 2 ) } + 1 ) ( x - \sqrt{( 2 ) } ) + \sqrt{( 2 ) } = 0\)

Discussione e risoluzione di equazioni numeriche frazionarie

Determina l’ I nsieme * *S oluzione delle seguenti equazioni frazionarie:

  1. \(\frac{3 }{x } - 2 = x\) ;;tab;;R. \(x _{1 } = - 3 \vee x _{2 } = 1\)

  2. \(\frac{4 - 3 x }{x } = \frac{3 - 2 x }{x ^{2 } }\) ;;tab;;R. \(x _{1 } = x _{2 } = 1\)

  3. \(\frac{1 }{x } = \frac{1 }{x + 1 } - 1\) ;;tab;;R. \(I.S. = \varnothing\)

  4. \(\frac{x }{2 } = \frac{x + 2 }{x - 2 } + 1\) ;;tab;;R. \(x _{1 } = 0 \vee x _{2 } = 6\)

  5. \(\frac{3 }{x - 1 } - \frac{1 }{x } + \frac{1 }{2 } = 0\) ;;tab;;R. \(x _{1 } = - 1 \vee x _{2 } = - 2\)

  6. \(\frac{3 x }{x ^{2 } - 9 } + \frac{x }{2 x - 6 } = 1\) ;;tab;;R. \(x _{1,2 } = \frac{9 \pm 3 \sqrt{17 } }{2 }\)

  7. \(\frac{x + 9 }{x - 3 } = 2 - \frac{x - 3 }{x + 9 }\) ;;tab;;R. \(I.S. = \varnothing\)

  8. \(\frac{x }{x + 1 } = \frac{4 }{x + 2 }\) ;;tab;;R. \(x _{1,2 } = 1 \pm \sqrt{5 }\)

  9. \(\frac{4 x - 3 }{x ^{2 } - 4 } - \frac{3 x }{x - 2 } = \frac{4 }{2 - x } - \frac{4 x }{2 + x }\) ;;tab;;R. \(x _{1 } = 1\) ; \(x _{2 } = 5\)

  10. \(\frac{3 x + 2 }{2 x ^{2 } - 2 x - 12 } - \frac{3 - x }{4 x - 12 } = - \frac{3 }{x + 2 }\) ;;tab;;R. \(x _{1 } = - 19\) ; \(x _{2 } = 2\)

  11. \(\frac{2 x + 1 }{x } = \frac{x }{2 x + 1 }\) ;;tab;;R. \(x _{1 } = - 1 \vee x _{2 } = - \frac{1 }{3 }\)

  12. \(\frac{4 - x }{18 - 2 x ^{2 } } + \frac{2 }{3 - x } = \frac{6 x }{4 x + 12 }\) ;;tab;;impossibile

  13. \(\frac{6 }{9 x ^{2 } - 12 x + 4 } + \frac{1 }{3 x - \frac{1 }{2 } } = 0\)

  14. \(x - 1 - \frac{1 }{x - 1 } = \frac{6 }{6 - 6 x }\) ;;tab;;R. \(I.S. = \varnothing\)

  15. \(\frac{6 x - 6 }{x ^{2 } - 4 x + 3 } + \frac{x ^{2 } - x - 6 }{x - 3 } = - 2\) ;;tab;;R. \(x _{1 } = - 3\) ; \(x _{2 } = 2\)

  16. \(\frac{x - 4 }{x - 2 } + \frac{x - 1 }{x ^{2 } - 5 x + 6 } - \frac{4 - 2 x }{3 - x } = 0\) ;;tab;;R. \(x = - 1\)

  17. \(\frac{x - 3 }{x - 1 } - \frac{4 }{3 } + \frac{x - 1 }{x + 1 } = 0\) ;;tab;;R. \(x _{1,2 } = 3 \pm \sqrt{10 }\)

  18. \(\frac{x - 1 }{x } + \frac{1 }{x + 1 } + \frac{2 + x }{x ^{2 } + x } = 0\) ;;tab;;R. \(x _{1 } = x _{2 } = - 1\)

  19. \(3 \left( x - \frac{1 }{3 } \right) + \frac{9 }{3x - 1 } = 10\)

  20. \(\frac{x + 1 }{\sqrt{2 } - x } = \frac{x - 2 }{x - 2 \sqrt{2 } }\) ;;tab;;R. \(x _{1 } = 0 ; x _{2 } = \frac{1 + 3 \sqrt{2 } }{2 }\)

  21. \(\frac{1 }{x ^{2 } + x - 2 } - \frac{1 }{x ^{3 } - 2 x ^{2 } + x } = \frac{1 }{3 x ^{2 } - 3 x }\) ;;tab;;R. \(x _{1 } = - \frac{1 }{2 } ; x _{2 } = 4\)

  22. \(\frac{1 }{2 x - 4 } - \frac{2 }{x + 1 } - \frac{1 }{x - 1 } = \frac{1 }{x ^{2 } - 3 x + 2 }\) ;;tab;;R. \(x = \frac{7 }{5 }\)

  23. \(\frac{2 x }{x ^{2 } + 2 x - 8 } - \frac{2 x + 7 }{x ^{2 } - 3 x - 4 } = 0\) ;;tab;;R. \(x _{1 } = - 2\) ; \(x _{2 } = \frac{28 }{17 }\)

  24. \(\frac{1 - x }{x ^{2 } - 4 x + 3 } - \frac{4 }{9 - x ^{2 } } + \frac{x - 3 }{x ^{2 } + 4 x + 3 } = - \frac{5 }{3 - x }\) ;;tab;; R. \(x _{1 } = - 5\) ; \(x _{2 } = - \frac{1 }{5 }\)

  25. \(\frac{4 x - 7 }{x + 2 } + \frac{1 - 6 x ^{2 } }{x ^{2 } - 5 x + 6 } = \frac{x }{2 x ^{2 } - 2 x - 12 } - 2\) ;;tab;;impossibile

  26. \(\frac{1 }{x - 2 } + \frac{2 }{( x - 2 ) ^{2 } } = \frac{3 }{( x - 2 ) ^{3 } }\) ;;tab;;R. \(x _{1 } = - 1\) ; \(x _{2 } = 3\)

  27. \(\frac{1 }{x + 3 } - \frac{5 ( x + 2 ) }{( x + 3 ) ^{2 } } = \frac{5 x - 1 }{( x + 3 ) ^{3 } }\) ;;tab;; R. \(x _{1 } = - 5\) ; \(x _{2 } = - 1\)

  28. \(\frac{3 }{( 3 x - 6 ) ^{2 } } - \frac{x ^{2 } - 4 }{( 3 x - 6 ) ^{4 } } = 0\) ;;tab;; R. \(x = \frac{28 }{13 }\)

  29. \(\frac{2 x }{x ^{2 } - 2 x + 1 } = \frac{- 7 }{3 x ^{2 } - 21 x + 18 } + \frac{2 x }{x ^{2 } - 3 x + 2 }\) ;;tab;; R. \(x _{1 } = - 14\) ; \(x _{2 } = - 1\)

  30. \(\frac{5 x - 3 }{x ^{2 } - 5 x } + \frac{2 }{x } = \frac{3 x }{x ^{2 } + 3 x } - \frac{2 }{x + 3 } - \frac{4 }{5 - x }\) ;;tab;; R. \(x _{1,2 } = \frac{- 1 \pm \sqrt{313 } }{4 }\)

  31. \(\frac{x - 9 }{4 x - x ^{2 } } - \frac{3 x + 2 }{2 - x } = \frac{x - 5 }{x + 2 } + \frac{2 x ^{4 } + 6 x ^{3 } }{x ( x - 4 ) ( x ^{2 } - 4 ) }\) ;;tab;; impossibile

  32. \(\frac{3 ( x + 1 ) }{x - 1 } = 1 - \frac{2 x - 3 }{x }\)

  33. \(\frac{3 - 3 x }{x ^{2 } - 1 } + \frac{8 x }{2 - 2 x } = 0\) ;;tab;;R. \(x _{1,2 } = \frac{- 7 \pm \sqrt{97 } }{8 }\)

  34. \(\frac{1 }{x ^{2 } - 9 } + \frac{2 }{x - 3 } + \frac{2 x }{3 x + 9 } - \frac{31 }{3 x ^{2 } - 27 } = \frac{1 }{3 }\) ;;tab;;R. \(x _{1 } = - 1 \vee x _{2 } = 1\)

  35. \(\frac{\frac{1 }{1 + x } - \frac{1 }{1 - x } }{\frac{2 }{x - 1 } + \frac{2 }{x + 1 } } = \frac{2 x }{1 - x } - \frac{2 x }{1 + x }\) ;;tab;;R. \(x _{1 } = - \frac{1 }{3 } \vee x _{2 } = \frac{1 }{3 }\)

  36. \(\frac{x + 1 }{x - 2 \sqrt{3 } } - \frac{1 - x }{x + 2 \sqrt{3 } } = \frac{x ^{2 } + 8 }{x ^{2 } - 12 }\) ;;tab;;R. \(x _{1 } = \sqrt{6 } - \sqrt{2 } \vee x _{2 } = \sqrt{2 } - \sqrt{6 }\)

  37. \(\frac{2 x + 1 }{1 + x } + \frac{5 }{1 - x } - \frac{2 }{x ^{2 } - 1 } = 0\)

  38. \(\left( \frac{x + 1 }{x } \right) ^{2 } - \frac{2 ( 3 x - 1 ) }{x ^{2 } } = 5\) ;;tab;;R. \(x _{1 } = - \frac{3 }{2 } \vee x _{2 } = \frac{1 }{2 }\)

  39. \(\frac{( x - 2 ) ^{2 } }{x ^{2 } - 1 } - \frac{x + 2 }{x + 1 } + \frac{x }{2 x + 2 } = 0\) ;;tab;;R. \(x _{1,2 } = \frac{11 \pm \sqrt{73 } }{2 }\)

  40. \(- \frac{x ^{2 } }{x + 2 } + \frac{2 x }{x - 2 } = - \frac{x + x ^{3 } }{x ^{2 } - 4 }\) ;;tab;;R. \(x _{1 } = 0 \vee x _{2 } = - \frac{5 }{4 }\)

  41. \(\frac{5 }{x + 1 } + \frac{2 x }{x - 2 } = \frac{6 x ^{2 } - 10 }{x ^{2 } - x - 2 }\) ;;tab;;R. \(x _{1 } = 0 \vee x _{2 } = \frac{7 }{4 }\)

  42. \(\frac{x + 1 }{x - 2 } - \frac{3 x }{x + 3 } = \frac{x ^{2 } + 2 x }{x ^{2 } + x - 6 }\) ;;tab;; R. \(x _{1 } = - \frac{1 }{3 } \vee x _{2 } = 3\)

  43. È vero che in \(\mathbb{R}\)

    \(\frac{3 }{1 + x ^{2 } } = \frac{3 }{x ^{4 } + 2 x ^{2 } + 1 }\) e \(\frac{2 x + 14 }{x ^{3 } - x ^{2 } + 4 x - 4 } - \frac{4 }{x - 1 } = \frac{2 }{x ^{2 } + 4 }\) sono equivalenti?

  44. Verifica che vale 1 il prodotto delle soluzioni dell’equazione \(\frac{x }{1 - x ^{3 } } + \frac{2 x - 2 }{x ^{2 } + x + 1 } = 0\) .

  45. Sull’asse reale rappresenta il D ominio e l’ I nsieme S oluzione dell’equazione \(\frac{x + 2 }{x } = 2 + \frac{x }{x + 2 }\) .

  46. Stabilisci se esiste qualche numero reale per cui la somma delle due frazioni \(f _{1 } = \frac{2 - x }{x + 2 } e f _{2 } = \frac{x + 1 }{x - 1 }\) è uguale a \(\frac{9 }{5 }\) .

  47. L’espressione \(E = \frac{4 x }{1 - x ^{2 } } + \frac{1 - x }{1 + x } - \frac{1 + x }{1 - x }\) non assume mai il valore –1. VERO o FALSO?

Discussione e risoluzione di equazioni letterali

Risolvi le seguenti equazioni letterali ed eventualmente discutile

  1. \(x ^{2 } - a x = 0\) ;;tab;;R. \(x _{1 } = 0 \vee x _{2 } = a\)

  2. \(x ^{2 } - a x = 0\) ;;tab;;R. \(a = 0 \rightarrow \mathbb{R} ; a \neq 0 \rightarrow x _{1 } = - 2 a \vee x _{2 } = 2 a\)

  3. \(x ^{2 } + ( x - a ) ^{2 } = 2 a x\) ;;tab;;R. \(x _{1 } = \frac{2 - \sqrt{2 } }{2 } a \vee x _{2 } = \frac{2 + \sqrt{2 } }{2 } a\)

  4. \(( 2 x - a ) x = a x\) ;;tab;;R. \(x _{1 } = 0 \vee x _{2 } = a\)

  5. \(x ^{2 } - a x - 6 a ^{2 } = 0\) ;;tab;;R. \(x _{1 } = - 2 a \vee x _{2 } = 3 a\)

  6. \(( a - 3 ) x ^{2 } - a x + 3 = 0\) ;;tab;;R. \(x _{1 } = 1 \vee x _{2 } = \frac{3 }{a - 3 }\)

  7. \(a x ^{2 } - a ^{2 } x + x ^{2 } + x - a x - a = 0\) ;;tab;;R. \(x _{1 } = a \vee x _{2 } = - \frac{1 }{a + 1 }\)

  8. \(\frac{x }{a } + \frac{x ^{2 } }{a - 1 } = 0\) ;;tab;;R. \(a \neq 0 \wedge a \neq 1 \rightarrow x _{1 } = 0 \vee x _{2 } = \frac{1 - a }{a }\)

  9. \(\frac{x }{a + 1 } + \frac{x ^{2 } }{a - 1 } = 0\) ;;tab;;R. \(a \neq - 1 \wedge a \neq 1 \rightarrow x _{1 } = 0 \vee x _{2 } = \frac{1 - a }{a + 1 }\)

  10. \(\frac{2 x }{3 + k x } - \frac{x }{3 - k x } = 0\) ;;tab;;R. \(x _{1 } = 0 \vee x _{2 } = \frac{1 }{k }\)

  11. \(\frac{m - n }{m n } x ^{2 } = \frac{2 m ^{2 } n }{m ^{2 } - n ^{2 } } - \frac{m n }{m + n }\) ;;tab;;R. \(x _{1,2 } = \frac{\pm m }{m - n }\)

  12. \(\frac{m x - x ^{2 } }{m ^{2 } - 3 m + 2 } - \frac{x }{2 - m } - \frac{m + 1 }{m - 1 } = 0\) ;;tab;;R. \(x _{1 } = m - 2 \vee x _{2 } = m + 1\)

  13. \(\frac{x ^{2 } + 2 t x }{t ^{2 } - t x } - 2 = \frac{3 t }{t - x } + \frac{x + t }{t }\) ;;tab;;R. \(x = - 3t\)

  14. \(\frac{x - 1 }{k + 1 } - \frac{x ^{2 } + 1 }{k ^{2 } - 1 } = \frac{2 k }{1 - k ^{2 } }\) ;;tab;;R. \(x _{1 } = - 1 ; x _{2 } = k\)

  15. \(2 \cdot \sqrt{m } - x = \frac{m - 1 }{x }\) ;;tab;;R. \(x _{1,2 } = \sqrt{m } \pm 1\)

  16. E’ vero che l’equazione \(1 - \frac{1 }{k + x } - \frac{1 }{k - x } = 0\) ammette due soluzioni reali coincidenti se \(k = 2\) ?

  17. Nell’equazione \(( a - 1 ) \cdot ( x + a ) = \frac{x + a }{x - 1 } \cdot [ x ( a + 1 ) - 2 a ]\) , dopo aver completato la discussione, stabilisci per quali valori di a le radici che si ottengono dall’equazione completa sono entrambe positive.

  18. È vero che l’equazione \(3 k x ^{2 } + ( x - k ) ^{2 } + 2 k ( k + x ) = 0\) ammette radici reali opposte se \(k < - \frac{1 }{3 }\) ?

  19. Per quali valori di b l’equazione \(\frac{5 x ^{2 } - 4 ( b + 1 ) }{b ^{2 } - 4 } - \frac{3 x - 1 }{b + 2 } = \frac{3 - 2 x }{2 - b } - \frac{3 x }{b ^{2 } - 4 }\) ha una soluzione negativa?

  20. Per l’equazione \(( x - k - 1 ) ^{2 } = ( k + 1 ) \cdot ( k - 2 x + x ^{2 } )\) , completate le implicazioni:

    1. \(k = 0\) equazione …................. \(I.S. = \ldots \ldots \ldots \ldots \ldots \ldots \ldots\)
    2. \(k = - 1\) equazione …..................... \(x _{1 } = \ldots \ldots \ldots \ldots \ldots \ldots \ldots\)
    3. \(k = \ldots \ldots\) equazione pura; due soluzioni reali …....................... se …................ \(x _{1 } = \ldots \ldots \ldots \vee x _{2 } = \ldots \ldots \ldots\)
  21. Stabilisci per quali valori del parametro m l’equazione \(\frac{m + 2 }{x - 2 } + m x = 2\) ammette soluzioni reali distinte. Se \(m = - 2\) sono accettabili le radici reali trovate?

  22. Dopo aver completato la discussione dell’equazione parametrica \(\frac{x + 1 }{b - 1 } + \frac{b - 1 }{x + 1 } = \frac{3 x ^{2 } + 2 - b x }{b x + b - 1 - x }\) , determina per quale valore del parametro le soluzioni sono accettabili.

  23. Le soluzioni dell’equazione \(( x + b ) ^{2 } = ( b + 1 ) ^{2 }\) con \(b \neq - 1\) sono:

    Tabella6
    [A] \(x _{1 } = - 1 ; x _{2 } = 1\) [B] \(x _{1 } = - 2 b - 1 ; x _{2 } = 1\) [C] \(x _{1 } = x _{2 } = 1\) [D] \(x _{1 } = 1 - 2 b ; x _{2 } = 1\)
  24. Per quali valori di k l’equazione \(x ^{2 } - ( 2 k + 1 ) x + 3 k + 1 = 0\) ammette soluzioni reali coincidenti?

Relazioni tra soluzioni e coefficienti

  1. Per ciascuna delle seguenti equazioni * scrivi il discriminante, se l’equazione ha soluzioni reali, la somma e il prodotto delle soluzioni* :
    1. \(5 x ^{2 } + 2 x - 1 = 0\)
    2. \(- 3 x ^{2 } + 1 = 0\)
    3. \(6 x ^{2 } + 7 x = 0\)
    4. \(- x ^{2 } + x - 1 = 0\)
    5. \(x ^{2 } + 2 x + 1 = 0\)
    6. \(2 x ^{2 } - 7 x + 1 = 0\)
  2. Senza risolvere le equazioni determina somma e prodotto dello loro radici
    1. \(x ^{2 } + 4ax + a = 0\)
    2. \(2x ^{2 } - \sqrt{2 } x + 1 = 0\)
    3. \(2x ^{2 } + 6kx + 3k ^{2 } = 0\)
    4. \(3 \sqrt{3 } x ^{2 } - 6 \sqrt{3 } x + 2 = 0\)
    5. \(\sqrt{2 } x ^{2 } + ( \sqrt{3 } - \sqrt{2 } ) x + 4 = 0\)
    6. \(( \sqrt{5 } + \sqrt{3 } ) x ^{2 } - ( \sqrt{5 } - \sqrt{3 } ) x + 1 = 0\)
  3. Dell’equazione \(3 \sqrt{2 } x ^{2 } - 5 x + \sqrt{2 } = 0\) è nota la radice \(x _{1 } = \frac{1 }{\sqrt{2 } }\) ; senza risolvere l’equazione determinare l’altra radice.
  4. Si determini la relazione che lega i coefficienti della generica equazione di secondo grado alla somma dei quadrati delle radici. Si vuole esprimere, attraverso i coefficiente a , b , c dell’equazione la quantità \(x _{1 } ^{2 } + x _{2 } ^{2 }\) . Si tenga presente la seguente identità \(x _{1 } ^{2 } + x _{2 } ^{2 } = \left( x _{1 } + x _{2 } \right) ^{2 } - 2 x _{1 } x _{2 }\) .
  5. Senza risolvere le equazioni stabilisci quale ha come soluzioni due numeri reali positivi e quale due numeri reali reciproci: \(e _{1 } \text{ :} 5 x ^{2 } + 2 x - 1 = 0 \text{ ;} e _{2 } \text{ :} - x ^{2 } + x - 1 = 0 \text{ ;} e _{3 } \text{ :} 2 x ^{2 } - 7 x + 1 = 0\)
  6. Un’equazione di secondo grado ha il primo coefficiente uguale a \(- \frac{3 }{2 }\) ; sapendo che l’insieme soluzione è \(I.S. = \left\{ - \frac{3 }{4 } ; \sqrt{2 } \right\}\) determinate i suoi coefficienti b e c.
  7. Dell’equazione \(a x ^{2 } + b x + c = 0\) la somma delle soluzioni è \(\frac{21 }{5 }\) e una soluzione è \(x _{1 } = 3,2\) ; determinate \(x _{2 }\) .
  8. Determinate i coefficienti a, b, c di un’equazione di secondo grado sapendo che \(x _{1 } = 1 - \sqrt{2 }\) , il prodotto delle soluzioni è \(- 1\) e la somma del secondo con il terzo coefficiente è \(9\) .
  9. Determinate i coefficienti b e c dell’equazione \(x ^{2 } + b x + c = 0\) sapendo che una radice è tripla dell’altra e la loro somma è 20.
  10. Dopo aver completato la discussione dell’equazione parametrica \(\frac{x + 1 }{b - 1 } + \frac{b - 1 }{x + 1 } = \frac{3 x ^{2 } + 2 - b x }{b x + b - 1 - x }\) , determina se esiste qualche valore del parametro per cui \(x _{1 } + x _{2 } = x _{1 } \cdot x _{2 }\) .

Determinare due numeri conoscendone la somma e il prodotto

  1. Determina, se possibile, due numeri aventi somma e prodotto indicati
    1. \(S = 3 ; P = 5\)
    2. \(S = 7 ; P = 2\)
    3. \(S = - 3 ; P = - 8\)
    4. \(S = - 5 ; P = 4\)
    5. \(S = \frac{1 }{2 } ; P = \frac{2 }{3 }\)
    6. \(S = \sqrt{2 } ; P = 2\)
    7. \(S = \sqrt{7 } - 1 ; P = 6\)
    8. \(S = a + 1 ; P = a ^{2 }\)
  2. Scrivi un’equazione di secondo grado che ammette come radici le soluzioni indicate
    1. \(x _{1 } = - 2 ; x _{2 } = 5\)
    2. \(x _{1 } = 7 ; x _{2 } = 2\)
    3. \(x _{1 } = - \frac{1 }{2 } ; x _{2 } = \frac{3 }{4 }\)
    4. \(x _{1 } = \frac{2 }{3 } ; x _{2 } = \frac{1 }{3 }\)
    5. \(x _{1 } = \sqrt{2 } ; x _{2 } = \sqrt{5 }\)
    6. \(x _{1 } = \frac{1 + \sqrt{2 } }{2 } ; x _{2 } = \frac{1 - \sqrt{2 } }{2 }\)
  3. Nell’equazione \(2 x ^{2 } + 6 k x + 3 k ^{2 } = 0\) determinare i valori di \(k\) per cui tra le radici reali distinti sussista la relazione \(x _{1 } + x _{2 } = x _{1 } \cdot x _{2 }\) .
  4. Determinate il perimetro del rombo avente \(area = 24 m ^{2 }\) , sapendo che la somma delle misure delle sue diagonali è \(14 m\) .
  5. Costruire i due triangoli isosceli aventi \(area = 120 m ^{2 }\) sapendo che \(31 m\) è la somma delle misure della base con l’altezza.
  6. Il triangolo rettangolo ABC ha l’ipotenusa AC di \(40 cm\) e l’altezza BH ad essa relativa di \(19,2 cm\) . Determinate la misura delle proiezioni dei cateti sull’ipotenusa.

Scomposizione del trinomio di secondo grado

  1. Scrivere un’equazione di secondo grado che ammetta le soluzioni \(x _{1 } = \frac{1 }{2 } \text{ e } x _{2 } = 3\) .

    In virtù di quanto visto in questo paragrafo, si ha: \(\left( x - \frac{1 }{2 } \right) \left( x + 3 \right) = 0\) da cui: \(x ^{2 } + 3 x - \frac{1 }{2 } x – \frac{3 }{2 } = 0\) cioè: \(x ^{2 } + 5 x – \frac{3 }{2 } = 0\) ovvero: \(2 x ^{2 } + 5 x - 3 = 0\)

Scomponi in fattori i seguenti trinomi di secondo grado

  1. \(x ^{2 } - 5 x - 14\)

    ;;tab;;R .

    \(( x + 2 ) ( x - 7 )\)

  2. \(2 x ^{2 } + 6 x - 8\) ;;tab;;R. \(2 ( x - 1 ) ( x + 4 )\)

  3. \(- 3 x ^{2 } + \frac{39 }{2 } x - 9\) ;;tab;;R. \(- 3 \left( x - \frac{1 }{2 } \right) ( x - 6 )\)

  4. \(- 2 x ^{2 } + 7 x + 4\)

  5. \(4 x ^{2 } + 4 x - 15\) ;;tab;;R. \(4 \left( x - \frac{3 }{2 } \right) \left( x + \frac{5 }{2 } \right)\)

  6. \(3 x ^{2 } + 3 x - 6\) ;;tab;;

  7. \(4 x ^{2 } - 9 x + 2\) ;;tab;;R. \(4 ( x - 2 ) \left( x - \frac{1 }{4 } \right)\)

  8. \(2 x ^{2 } + 2 x – \frac{3 }{2 }\) ;;tab;;

  9. \(3 x ^{2 } + 5 x – 2\) ;;tab;;R. \(3 \left( x - \frac{1 }{3 } \right) ( x + 2 )\)

  10. \(4 x ^{2 } - 24 x + 20\)

  11. \(2 x ^{2 } - \frac{4 }{3 } x – \frac{16 }{3 }\) ;;tab;;R. \(2 ( x - 2 ) \left( x + \frac{4 }{3 } \right)\)

  12. \(\frac{4 }{3 } x ^{2 } + \frac{11 }{3 } x – \frac{7 }{2 }\)

  13. \(3 x ^{2 } - 6 x - 12\) ;;tab;;R. \(3 \left( x - 1 - \sqrt{5 } \right) \left( x - 1 + \sqrt{5 } \right)\)

  14. \(2 x ^{2 } - 8 x + 2\) ;;tab;;

  15. \(- \frac{1 }{2 } x ^{2 } + x + \frac{3 }{8 }\) ;;tab;;R. \(- \frac{1 }{2 } \left( x - 1 - \frac{\sqrt{7 } }{2 } \right) \left( x - 1 + \frac{\sqrt{7 } }{2 } \right)\)

  16. \(- \frac{3 }{4 } x ^{2 } - \frac{9 }{2 } x – \frac{45 }{8 }\) ;;tab;;R. \(- \frac{3 }{4 } \left( x + 3 - \frac{\sqrt{6 } }{2 } \right) \left( x + 3 + \frac{\sqrt{6 } }{2 } \right)\)

Regola di Cartesio

*Determina il segno delle soluzioni di ogni equazione senza risolverla, dopo aver verificato che * \(\Delta \geq 0\)

  1. \(x ^{2 } - 5 x + 6 = 0\)
  2. \(- x ^{2 } + x + 42 = 0\)
  3. \(x ^{2 } + x - 20 = 0\)
  4. \(3 x ^{2 } + 2 x - 1 = 0\)
  5. \(2 x ^{2 } - \sqrt{5 } x - 1 = 0\)
  6. \(3 x ^{2 } + 5 x + 1 = 0\)
  7. \(- x ^{2 } - x + 1 = 0\)
  8. \(- 5 x + 1 - x ^{2 } = 0\)
  9. \(- 1 - x ^{2 } - 2 x = 0\)
  10. \(1 + x + 2 x ^{2 } = 0\)
  11. \(x ^{2 } - 4 \sqrt{2 } x + 2 = 0\)
  12. \(- \frac{1 }{2 } x ^{2 } + x + \frac{3 }{8 } = 0\)

Equazioni parametriche

  1. Assegnata l’equazione \(( 1 - k ) x ^{2 } + ( k - 2 ) x + 1 = 0\) , stabilire i valori da assegnare al parametro \(k\) affinché le soluzioni reali distinte abbiano la somma positiva.

    Svolgimento guidato

    Nel testo del problema vi sono due richieste: a) le soluzioni siano reali distinte e b) abbiano somma positiva.

    Il problema si formalizza attraverso il sistema \(\{ \begin{array}{l } \Delta > 0 \\- \frac{b }{a } > 0 \end{array} \rightarrow \{ \begin{array}{l } ( k - 2 ) ^{2 } - 4 ( 1 - k ) > 0 \\- \frac{k - 2 }{1 - k } > 0 \end{array}\) ; risolviamo la prima disequazione: \(d _{1 } \ldots \ldots \ldots \ldots \ldots \ldots > 0 \rightarrow k ^{2 } > 0 \rightarrow I.S. _{1 } = \left \{ \right .k \in \mathbb{R} \text{ |} k \neq 0 \}\) e la seconda disequazione studiando il segno del numeratore e del denominatore: \(\{ \begin{array}{l } N \div - k + 2 > 0 \rightarrow k < 2 \\D \div 1 - k > 0 \rightarrow k < 1 \end{array}\) da cui con la tabella dei segni ricaviamo \(I.S. _{2 } = \{ k \in \mathbb{R} \text{ |} k \ldots \ldots \ldots \vee k > \ldots \ldots \ldots \}\) .

    _images/immagini14.png

    Dal grafico si ricava \(I.S. = I.S. _{1 } \cap I.S. _{2 } = \{ k \in \mathbb{R} \text{ |} k \ldots \ldots \ldots \vee 0 < k < \ldots \ldots \vee k \ldots \ldots \ldots \}\)

  2. Assegnata l’equazione \(( k + 1 ) x ^{2 } + ( k + 3 ) x + k = 0\) stabilire per quale valore di \(k\) una sua soluzione è \(x = - 1\) . In tale caso determinare l’altra soluzione.

    Traccia di svolgimento

    Ricordiamo che un valore numerico è soluzione di un’equazione se sostituito all’incognita trasforma l’equazione in una uguaglianza vera. Per questo motivo, sostituendo all’incognita il valore assegnato, il parametro \(k\) dovrà verificare l’uguaglianza: \(( k + 1 ) ( - 1 ) ^{2 } + ( k + 3 ) ( - 1 ) + k = 0 \rightarrow ..................\)

    Sostituendo il valore di k trovato, l’equazione diventa: \(3 x ^{2 } + 5 x + 2 = 0\) ; l’altra soluzione può essere trovata o con la formula risolutiva, oppure ricordando che \(x _{1 } + x _{2 } = - \frac{b }{a } = - \frac{5 }{3 } \rightarrow x _{2 } = .......\) o anche \(x _{1 } \cdot x _{2 } = \frac{c }{a } = \frac{2 }{3 } \rightarrow x _{2 } = ............\) .

  3. Giustificare la verità della seguente proposizione: “per qualunque valore assegnato al parametro \(m\) l’equazione \(( m - 1 ) x ^{2 } + 2 m x + m + 1 = 0\) ha soluzioni reali distinte”.

  4. Determinare \(m\) affinché: \(a ) x _{1 } + x _{2 } = 1 - \sqrt{3 }\) ; \(b ) x _{1 } \cdot x _{2 } = \frac{12 }{5 }\) ; \(c ) x _{1 } + x _{2 } = 1 - x _{1 } \cdot x _{2 }\)

  5. Nell’equazione \(7 x ^{2 } + ( k - 5 ) x - ( k + 2 ) = 0\) determinare \(k\) affinché le soluzioni siano reali; distingui i casi “reali coincidenti” e “reali distinte”.

    Nel primo caso determina \(x _{1 } = x _{2 } = \ldots \ldots \ldots \ldots \ldots\) ; nel secondo caso, determina \(k\) affinché

    1. il prodotto delle soluzioni sia \(- 8 / 3\) ;
    2. una soluzione sia nulla;
    3. le soluzione siano una il reciproco dell’altra, cioè: \(x _{1 } = 1 / x _{2 }\) ;
    4. la somma dei reciproci delle soluzioni sia \(1 / 2\) ;
    5. la somma delle soluzioni superi il loro prodotto di \(2\) .
  6. Verificare che nell’equazione \(( 2 m - 3 ) x ^{2 } - ( m + 2 ) x + 3 m - 2 = 0\) si hanno due valori del parametro per cui le soluzioni sono reali coincidenti. Determina i due valori.

  7. Nell’equazione \(x ^{2 } - 2 ( k + 2 ) x + ( k ^{2 } - 3 k + 2 ) = 0\) determinare \(k\) affinché le soluzioni siano reali, con somma positiva e prodotto negativo.

    *Traccia di svolgimento: *

    Il problema richiede tre condizioni alle quali deve soddisfare contemporaneamente il parametro, pertanto si formalizza con il sistema \(\left \{ \begin{array}{l } \Delta \geq 0 \\- \frac{b }{a } > 0 \\\frac{c }{a } < 0 \end{array}\right .\)

  8. \(x ^{2 } - 2 x - k = 0\) determinare \(k\) in modo che

    1. le soluzioni siano reali e distinte (Δ>0);;tab;;R. \([ k > - 1 ]\)
    2. la somma delle soluzioni sia 10 \(( x _{1 } + x _{2 } = 10 )\) ;;tab;;impossibile
    3. il prodotto delle soluzioni sia 10 \(( x _{1 } \cdot x _{2 } = 10 )\) ;;tab;;R. \([ k = - 10 ]\)
    4. una soluzione sia uguale a 0 (sostituire 0 alla x);;tab;;R. \([ k = 0 ]\)
    5. le radici siano opposte \(( x _{1 } + x _{2 } = 0 )\) ;;tab;;impossibile
    6. le radici siano reciproche \(( x _{1 } \cdot x _{2 } = 1 )\) ;;tab;;R. \([ k = - 1 ]\)
    7. le radici siano coincidenti (Δ=0);;tab;;R. \([ k = - 1 ]\)
    8. la somma dei quadrati delle radici sia 12 \(( x _{1 }^{2 } + x _{2 }^{2 } = ( x _{1 } + x _{2 } ) ^{2 } - 2x _{1 } x _{2 } = 12 )\) ;;tab;;R. \([ k = 4 ]\)
    9. la somma dei reciproci delle radici sia -4 \(\left( \frac{1 }{x _{1 } } + \frac{1 }{x _{2 } } = \frac{x _{1 } + x _{2 } }{x _{1 } x _{2 } } = - 4 \right)\) ;;tab;;R. \(\left[ k = \frac{1 }{2 } \right]\)
    1. la somma dei cubi delle radici sia 1 \(x _{1 }^{3 } + x _{2 }^{3 } = ( x _{1 } + x _{2 } ) ^{3 } - 3x _{1 }^{2 } x _{2 } - 3x _{1 } x _{2 }^{2 } = ( x _{1 } + x _{2 } ) ^{3 } - 3x _{1 } x _{2 } ( x _{1 } + x _{2 } ) = 1\) ;;tab;;R. \(\left[ k = - \frac{7 }{6 } \right]\)
    2. le radici siano entrambe negative \(\{ \begin{array}{l } x _{1 } \cdot x _{2 } > 0 \\x _{1 } + x _{2 } < 0 \end{array}\) ;;tab;;R. impossibile
  9. \(x ^{2 } - k x - 1 = 0\) determinare \(k\) in modo che

    1. le soluzioni siano coincidenti;;tab;;impossibile
    2. la somma delle radici sia 8;;tab;; R. \([ k = 8 ]\)
    3. le radici siano opposte;;tab;;R. \([ k = 0 ]\)
    4. una radice sia \(- \frac{1 }{3 }\) ;;tab;;R. \(\left[ k = \frac{8 }{3 } \right]\)
    5. i l prodotto delle radici sia -1;;tab;;R. \([ \forall k \in \mathbb{R} ]\)
  10. \(x ^{2 } + ( k + 1 ) x + k = 0\) determinate \(k\) affinché l’equazione

    1. abbia una soluzione sia uguale a zero;;tab;; R. \([ k = 0 ]\)
    2. abbia soluzioni opposte;;tab;; R. \([ k = - 1 ]\)
    3. non abbia soluzioni reali;;tab;;impossibile
    4. abbia le radici siano reciproche;;tab;; R. \([ k = 1 ]\)
    5. abbia le radici siano positive (regola di Cartesio);;tab;;R. impossibile
  11. \(x ^{2 } - kx + 6 = 0\) determinate \(k\) affinché l’equazione

    1. abbia la somma delle radici uguale a 7;;tab;;R. \([ k = 7 ]\)
    2. abbia le radici reali e opposte;;tab;; R. impossibile
    3. abbia l a somma dei reciproci delle radici uguale a -6;;tab;;R. \([ k = - 36 ]\) ;;tab;;
    4. abbia una radice uguale a \(- \frac{3 }{2 }\) ;;tab;;R. \(\left[ k = - \frac{11 }{2 } \right]\)
  12. \(x ^{2 } + ( k + 1 ) x + k ^{2 } = 0\) determinare \(k\) affinché l’equazione

    1. abbia come soluzione -1;;tab;;R. \(k = 0 \vee k = 1\)
    2. abbia una soluzione doppia (x 1 =x 2 );;tab;;R. \(k = - \frac{1 }{3 } \vee k = 1\)
    3. abbia le radici reciproche;;tab;;R. \([ k = \pm 1 ]\)
    4. abbia una radice l’opposto della reciproca dell’altra;;tab;;impossibile
    5. abbia una radice nulla;;tab;;R. [k=0]
  13. \(kx ^{2 } - 2kx + k - 2 = 0\) determinare \(k\) affinché l’equazione

    1. abbia una radice nulla;;tab;;R. \([ k = 2 ]\)
    2. abbia la somma dei reciproci delle radici uguale a 1;;tab;;R. \([ k = - 2 ]\)
    3. abbia la somma dei quadrati delle radici uguale a 4;;tab;;R. \([ k = 2 ]\)
    4. abbia la somma delle radici che superi di 5 il loro prodotto;;tab;;R. \(\left[ k = \frac{1 }{2 } \right]\)
  14. \(x ( x - a ) = \frac{a + x }{a + 2 }\) determinate \(a\) affinché

    1. una soluzione sia 1;;tab;;R. \([ a = - 1 \pm \sqrt{2 } ]\)
    2. l’equazione sia di primo grado;;tab;;R. impossibile
    3. una soluzione sia uguale al reciproco dell’altra;;tab;;R. \([ a = - 1 ]\)
    4. la somma delle soluzioni sia il doppio del loro prodotto ;;tab;;R. \(a _{1.2 } = \left[ \frac{- 2 \pm \sqrt{3 } }{2 } \right]\)
    5. la somma dei quadrati delle soluzioni sia 0;;tab;;R.impossibile
    6. la somma delle radici sia l’opposto del loro prodotto;;tab;;R. impossibile
    7. le soluzioni siano reali e distinte
    8. l’equazione sia spuria
    9. la somma dei cubi delle soluzioni sia nulla
    10. le soluzioni siano reali e discordi
    11. la somma dei reciproci dei cubi sia 1
  15. \(kx ^{2 } - ( 2k + 1 ) x + k - 5 = 0\) determinare il valore di k per il quale

    1. l’equazione ha soluzioni reali;;tab;;R. \(k \geq - \frac{1 }{24 }\)
    2. il prodotto delle radici sia -2;;tab;;R. \(k = \frac{5 }{3 }\)
    3. la somma delle radici sia 1;;tab;;R. k=-1 non accettabile
    4. una soluzione sia -2;;tab;;R. \(k = \frac{1 }{3 }\)
    5. le soluzioni siano opposte;;tab;;R. \(k = - \frac{1 }{2 }\) non accettabile
    6. la somma dei reciproci sia 3;;tab;;R. \(k = 16\)
    7. le soluzioni siano reciproche;;tab;;impossibile
    8. una soluzione sia l’opposto del reciproco dell’altra
    9. la somma dei quadrati delle soluzioni sia 4;;tab;;R. \(k = \frac{( 7 \pm \sqrt{( 51 ) } ) }{2 }\)
    10. le radici siano concordi ;;tab;;R. \(- \frac{1 }{24 } \leq k < 0 \vee k > 5\)
    11. le radici siano entrambe negative;;tab;;R. \(- \frac{1 }{24 } \leq k < 0\)
    12. la somma delle radici uguagli l’opposto del loro prodotto
  16. Per quale valore di \(k \in \mathbb{R}\) l’equazione \(kx ^{2 } - x + k = 0\) non ammette soluzioni reali?

    [A] \(k \leq - \frac{1 }{2 } \vee k \geq + \frac{1 }{2 }\) [B] \(- \frac{1 }{2 } < k < \frac{1 }{2 }\) [C] \(k < - \frac{1 }{2 } \vee k > \frac{1 }{2 }\) [D] \(- \frac{1 }{2 } \leq k \leq \frac{1 }{2 }\)

  17. Per quale valore di \(k \in \mathbb{R}\) l’equazione \(x ^{2 } + ( k - 2 ) x + 1 = 0\) ammette due soluzioni reali e distinte?

    [A] \(k > 4\) [B] \(k = 0 \vee k = 4\) [C] \(0 < k < 4\) [D] \(k < 0 \vee k > 4\)

  18. Per quale valore di k l’equazione \(( k - 1 ) x ^{2 } + kx + ( k + 1 ) = 0\) ha una soluzione nulla?

    [A] \(k = 1\) [B] \(k = - 1\) [C] \(k = 0\) [D] nessun valore di k

  19. Per quale valore di k l’equazione \(kx ^{2 } + \frac{1 }{2 } x + 1 = 0\) ha due soluzioni identiche?

    [A] \(k = \frac{1 }{4 }\) [B] \(k = \frac{1 }{16 }\) [C] \(k = 2\) [D] nessun valore di k

  20. Per quale valore di k l’equazione \(( k + 3 ) x ^{2 } - 2x + k = 0\) ammette due soluzioni reciproche?

    [A] \(k = 0\) [B] \(k = - 3\) [C] qualsiasi [D] nessun valore di k

  21. Per quale valore di k l’equazione \(( k + 1 ) x ^{2 } - kx - 4 = 0\) ha una soluzione uguale a 2?

    [A] \(k = 4\) [B] \(k = - 2\) [C] \(k = 0\) [D] \(k = - 1\)

  22. Se l’equazione \(( k + 1 ) x ^{2 } - kx - 4 = 0\) ha una soluzione uguale a 2 quanto vale l’altra soluzione?

    [A] \(x = 0\) [B] \(x = - 2\) [C] \(x = \frac{1 }{2 }\) [D] \(x = 2\)

Problemi di secondo grado in una incognita

  1. Il quadrato di un numero reale supera la metà del numero stesso di 5. Determina i numeri reali che rendono vera la proposizione enunciata. [-2; 5/2]

  2. Il prodotto della metà di un numero relativo con il suo successivo è 666. Quali numeri verificano questa proprietà? [36; -37]

  3. Trova un numero positivo che addizionato al proprio quadrato dia come somma 156.

  4. Un numero addizionato al quadrato della sua metà, dà come risultato 120. Trova il numero.

  5. Verifica che non esiste alcun numero reale tale che il quadrato del suo doppio uguagli la differenza tra il triplo del suo quadrato e il quadrato della somma del numero con 3.

  6. Due numeri naturali hanno rapporto 2/3 e somma dei loro quadrati 3757. Individua i numeri che verificano questa proprietà. [51, 34]

  7. La somma dei quadrati di due numeri pari consecutivi è 580. Quali sono i due numeri?[16; 18]

  8. Di due numeri naturali consecutivi si sa che la somma dei loro reciproci è 9/20. Quali sono i due numeri? [4: 5]

  9. Di cinque numeri interi consecutivi si sa che la differenza tra il quadrato della somma degli ultimi due numeri e la somma dei quadrati dei primi tre è 702. Qual è il più piccolo di questi numeri? [17]

  10. La somma delle età di un padre con quella del figlio è 34. Sapendo che l’età del padre aumentata di 8 anni dà il quadrato dell’età del figlio, trovare le due età. [28, 6]

  11. Determina due numeri naturali sapendo che la somma tra il doppio del minore ed il triplo del maggiore è 42 e che il rapporto tra la loro somma e il loro prodotto è 5/12. [3, 12]

  12. Trova l’età di una persona sapendo che fra tre anni la sua età sarà uguale al quadrato della quinta parte dell’età che aveva tre anni fa. [33]

  13. Trova due numeri pari consecutivi tali che la somma del quadrato del minore con il loro prodotto sia 544. [16; 18]

  14. Trova due numeri naturali sapendo che il minore supera di 2 la terza parte del maggiore e che il quadrato del maggiore supera di 68 il quadrato del doppio del minore. [8; 18]

  15. Da un segmento di 25 cm ne vogliamo ottenere due in modo che la somma dei loro quadrati sia 337. [9; 16]

  16. In una frazione il numeratore e il denominatore hanno somma 14, mentre la somma dei loro quadrati è 106. Qual è la frazione? [5/9 e 9/5]

  17. Due navi partono contemporaneamente da uno stesso porto e arrivano alla stessa destinazione dopo aver percorso sulla stessa rotta a velocità costante 720 miglia. Sapendo che una delle due navi viaggia con una velocità di 1 nodo (1 miglio all’ora) superiore a quella dell’altra nave e che perciò arriva 3 ore prima a destinazione, determina le velocità in nodi delle due navi. [15; 16]

  18. Due navi che viaggiano su rotte perpen-dicolari a velocità costante si incontrano in mare aperto. Sapendo che una delle navi viaggia a 15 nodi (1 nodo = 1 miglio all’ora), dopo quanto tempo le due navi si trovano alla distanza di 40 miglia?

  19. Luca e Carlo bevono due aranciate in bottiglia. Nel tempo in cui Luca beve 11 sorsi, Carlo ne beve 8, ma due sorsi di Carlo equivalgono a tre di Luca. Quando Carlo inizia a bere Luca ha già preso 4 sorsi. Dopo quanti sorsi di Carlo le due bibite hanno lo stesso livello?

  20. Un maratoneta durante un allenamento fa due giri di un percorso di 22 km mantenendo in ciascun giro una velocità costante ma nel secondo giro la velocità è inferiore di 0,5 km/h rispetto al primo giro. A quali velocità ha corso se ha impiegato complessivamente 2 ore e un quarto?

  21. Un capitale di 1200 € è depositato in banca a un certo tasso di interesse annuale. Alla scadenza del primo anno gli interessi maturati vengono ridepositati sullo stesso conto. Alla scadenza del secondo anno si ritira la somma di 12854,70 euro. Qual è stato il tasso di interesse? [3,5%]

  22. In un rettangolo, se si aumenta di 2 metri la base e si riduce di un metro l’altezza, la sua area aumenta di 4 metri quadrati. Se invece si riduce di un metro la base e si aumenta di 2 metri l’altezza, l’area aumenta di 22 metri quadrati. Quali sono le dimensioni del rettangolo?

  23. Una ditta spende mensilmente 73500 in stipendi per i propri dipendenti. Aumentando di 5 il numero dei dipendenti, ma riducendo l’orario di lavoro, diminuisce a ciascuno lo stipendio di 200 e spende solamente 2500 in più per gli stipendi. Quanti dipendenti aveva inizialmente la ditta e quanto guadagnava ognuno di essi? [35, 2100]

  24. Da un cartoncino rettangolare (ABCD, come in figura) si vuole ritagliare un quadrato (DEFG) in modo che le due parti ottenute siano equivalenti. Determinare la misura del lato del quadrato sapendo che \(\overline {EC } = 6 cm \text{ e } \overline {AG } = 4 cm\) . \([ \overline {DE } = 12cm ]\)

  25. Un terreno a forma rettangolare di 6016m2 viene recintato con un muro lungo 350m. Quali sono le dimensioni del rettangolo? [47; 128]

  26. Determinare sul segmento AB di misura 5m un punto P tale che il rettangolo delle due parti sia equivalente al quadrato di lato 2m. Rappresenta con un disegno le soluzioni. [1cm; 4cm]

  27. Calcolare perimetro e area del triangolo ABC isoscele sulla base AB sapendo che la differenza tra la base e l’altezza ad essa relativa è .0,5m e tale è anche la differenza tra il lato CB e la base stessa. [2p=25m; A=30m2 ]

  28. La superficie del rettangolo ABCD supera di 119m2 la superficie del quadrato costruito sul lato minore AD. Determinare il perimetro e la misura della diagonale sapendo che i 7/10 del lato maggiore AB sono uguali ai 12/5 del lato minore. [2p=62m; d=25m]

  29. Nel trapezio rettangolo ABCD, il rapporto tra la base maggiore AB e la base minore CD è 8/5, il lato obliquo forma con AB un angolo di 45°. Determinare il perimetro sapendo che l’area è 312 m2. \([ 2p = 64 + 12 \sqrt{2 } ]\)

  30. Determina il perimetro di un rombo che ha l’area di 24m2 e il rapporto tra le diagonali 4/3. [40m]

  31. Un rettangolo ABCD ha il perimetro di 48cm e l’area di 128cm2. A una certa distanza x dal vertice A sui due lati AD e AB si prendono rispettivamente i punti P e Q. Alla stessa distanza x dal vertice C sui lati CB e CD si prendono rispettivamente i punti R e S. Sapendo che il rapporto tra l’area del rettangolo ABCD e l’area del quadrilatero PQRS è 32/23 calcola la distanza x. [6cm]

  32. Un trapezio rettangolo ha la base minore di 9cm, l’altezza i 2/9 della base maggiore e l’area di \(20 + 9 \sqrt{2 } cm ^{2 }\) . Determina la misura della base maggiore. \([ 3 \sqrt{2 } ]\)

    _images/immagini3.png
  33. Da un quadrato di 32 cm di lato vengono ritagliati due triangoli rettangoli come descritti in figura dalla parte colorata. Calcola la misura di x, inferiore alla metà del lato del quadrato, in modo che l’area totale dei due triangoli evidenziati sia pari a 344 cm 2 . \(\left[ 4cm \right]\)

  34. Il rettangolo ABCD ha l’area di 240 cm 2 e l’altezza AD di 12 cm. Si vuole trasformare il rettangolo in un triangolo AEF allungando l’altezza di una quantità 3x e accorciando la base di una quantità x (vedi figura) in modo che il nuovo triangolo AEF abbia l’area di 162 cm 2 .

    [x=2; la soluzione x=14 non è accettabile]

    _images/immagini4.png
  35. Il rettangolo AEFG ha l’area di 768 cm 2 e l’altezza AG di 24 cm. Si vuole allungare l’altezza di una quantità x e accorciare la base di una quantità doppia 2x in modo da ottenere un secondo rettangolo ABCD che abbia l’area di 702 cm 2 . Determina la quantità x. [3cm]

  36. Il rettangolo ABCD ha l’area di 558 cm 2 e il lato DC di 18 cm. Lo si vuole trasformare in un nuovo rettangolo AEFG accorciando l’altezza di una quantità 5x e allungando la base di una quantità 4x in modo che il nuovo rettangolo AEFG che abbia l’area di 228 cm 2 . Determina la quantità x necessaria a compiere la trasformazione richiesta. [5]

  37. Un trapezio isoscele di area 144cm 2 ha la base maggiore che supera di 10cm la base minore che a sua volta supera di 10cm l’altezza. Determina il perimetro del trapezio.

  38. La piramide di Cheope è a base quadrata ed ha una superficie totale pari a 135700 m2. Sapendo che l’apotema della piramide misura 180 metri, si calcoli la lunghezza del lato di base. [230 m]

  39. Un container a forma di parallelepipedo a base quadrata ha una superficie totale pari a 210 m2. L’altezza è il doppio del lato di base diminuito di 2 metri. Trovare la lunghezza del lato di base. [5m]

Problemi con un parametro

  1. Sul prolungamento dei lati AB, BC, CD, DA del quadrato ABCD prendi rispettivamente i punti Q, R, S, P in modo che QB=RC=SD=PA. Dimostra che PQRS è un quadrato; nell’ipotesi che sia \(AB = 3m\) determina \(\overline {AP }\) in modo che l’area di PQRS sia k, con k reale positivo.

    _images/immagini7.png

    Svolgimento

    Per dimostrare che PQRS è un quadrato dobbiamo dimostrare che i lati sono congruenti e che gli angoli sono retti.

    Poni \(\overline {AP } = x \text{ con } x > 0\)

    \(Area _{PQRS } = \overline {PQ } ^{2 } = \overline {PA } ^{2 } + \overline {AQ } ^{2 }\) per il teorema di Pitagora.

    Verifica che si ottiene l’equazione risolvente \(2 x ^{2 } + 6 x + ( 9 - k ) = 0\) . Poiché vogliamo soluzioni reali positive, discuti l’equazione con il metodo di Cartesio. Il discriminante è \(\Delta = 36 - 8 ( 9 - k )\) Pertanto l’equazione ammette soluzioni reali per \(k \geq \frac{9 }{2 }\) . Dal segno dei coefficienti, essendo i primi due coefficienti positivi si ha una permanenza e quindi una radice negativa che non è accettabile. Per ottenere una soluzione positiva ci deve essere una variazione di segno negli ultimi due coefficienti, in altre parole \(9 - k\) deve essere negativo cioè \(9 - k < 0 \rightarrow k > 9\) . Pertanto il problema ha soluzioni per \(k > 9\) .

    _images/immagini9.png
  2. Nel trapezio rettangolo ABCD di base maggiore BC, la diagonale AC è bisettrice dell’angolo \(B \widehat {C } D\) . Posto \(\overline {AB } = 1m\) , determina la base maggiore in modo che sia 2k il perimetro del trapezio.

    Disegna la figura, i dati e l’obiettivo del problema.

    Traccia dello svolgimento

    Poniamo \(\overline {BC } = x\) . Dall’informazione che la diagonale AC è bisettrice dell’angolo \(B \hat {C } D\) , possiamo dimostrare che ADC è un triangolo isoscele sulla base AC. L’equazione risolvente sarà determinata dalla relazione tra i lati che esprime il perimetro del trapezio. Dobbiamo quindi esprimere \(\overline {DC }\) in funzione di x. * Traccia l’altezza *DH del triangolo isoscele ADC e dopo aver dimostrato la similitudine di ABC con DHC, osserva che si ha \(DC \div AC = HC \div BC\) poiché \(HC = \frac{1 }{2 } AC\) si ha \(\frac{1 }{2 } \overline {AC } ^{2 } = \overline {DC } \cdot \overline {BC }\) da cui si può ricavare la misura di DC: \(DC = \frac{1 }{2 } \frac{AC ^{2 } }{BC }\) . Calcola ora \(\overline {AC } ^{2 }\) , applicando il teorema di Pitagora al triangolo ABC. * :math:`DC = frac{1 + x ^{2 } }{2 x }` *. L’equazione parametrica risolvente è \(2 x ^{2 } + x \cdot ( 1 - 2 k ) + 1 = 0 \text{ con } x > 0\) ; può essere discussa con il metodo di Cartesio.

  3. Ad una sfera di raggio 1m è circoscritto un cono il cui volume è k volte il volume della sfera. Determina l’altezza del cono.

    _images/immagini1.png

    Dati

    \(\begin{array}{l } \overline {OC } = 1 \\\overline {OC } = \overline {OH } \\OC \perp VB \\\overline {BC } = \overline {BH } \\\overline {AH } = \overline {HB } \\VH \perp AB \\Volume _{cono } = k \cdot Volume _{sfera } \end{array}\)

    Obiettivo

    \(\text{ ?} \overline {VH }\)

    Poniamo \(\overline {VO } = x \text{ con } x > 0 \text{ da cui } \overline {VH } = \overline {VO } + \overline {OH } = x + 1\) .

    Ricordiamo che \(V _{cono } = \frac{1 }{3 } \pi \overline {HB } ^{2 } \cdot \overline {VH } \text{ e } V _{sfera } = \frac{4 }{3 } \pi \overline {CO } ^{3 }\) . Per impostare l’equazione risolvente dobbiamo cercare di esprimere \(\overline {HB } ^{2 }\) in funzione di x . Verifica che dalla similitudine di VOC con VHB si deduce: \(\overline {HB } \div \overline {OC } = \overline {VH } \div \overline {VC } \text{ quindi } \overline {HB } = \frac{\overline {OC } \cdot \overline {VH } }{\overline {VC } }\) ; dobbiamo ancora ricavare \(\overline {VC }\) che per il teorema di Pitagora su VCO è … Sostituendo tutti gli elementi trovati nella relazione che lega il volume del cono con il volume della sfera, verifica che si ottiene \(x ^{2 } + 2 x ( 1 - 2 k ) + 4 k = 0 \text{ con } x > 0\) , da discutere con il metodo di Cartesio.

  4. Il quadrilatero ABCD ha le diagonali perpendicolari ed è inscritto in una circonferenza; sapendo che \(\overline {AB } = 5 a ; \overline {AE } = 3 a ; 2 p _{BCA } = \frac{5 }{2 } \cdot \overline {BD }\) , essendo E punto d’incontro delle diagonali, determinate la misura delle diagonali.[Poni \(\overline {CE } = x\) ]

  5. Il rettangolo ABCD ha i lati AB e BC che misurano rispettivamente a e 3a (a>0). Prolunga il lato AB di due segmenti congruenti BN e AM e sia V il punto di intersezione delle retta MD e CN. Posto \(\overline {BN } = x\) , determina la misura della base MN del triangolo MVN in modo che la sua area sia k volte l’area del rettangolo assegnato.

  6. Due numeri reali hanno come somma a \(( a \in \mathbb{R} _{0 } )\) ; determinare i due numeri in modo che il loro prodotto sia k \(( k \in \mathbb{R} _{0 } )\) . Quale condizione si deve porre sull’incognita? Per quale valore del parametro i due numeri soluzione sono uguali?

  7. In un triangolo rettangolo l’altezza AH relativa all’ipotenusa BC misura 1m e \(A \hat {B } C = \text{ 60°}\) . Determinare sulla semiretta AH, esternamente al triangolo, un punto P in modo che sia k la somma dei quadrati delle distanze di P dai vertici del triangolo. Quale condizione va imposta al parametro k perché il problema abbia significato?

  8. \(\overline {AB } = 16 a ; \overline {BC } = 2 a \sqrt{14 }\) rappresentano le misure dei lati del rettangolo ABCD; determinare un punto P del segmento AB tale che la somma dei quadrati delle sue distanze dai vertici C e D sia uguale al quadrato della diagonale DB. Posto \(\overline {AP } = x\) quale delle seguenti condizioni deve rispettare la soluzione?. Dopo aver risolto il problema spiegare il significato delle soluzioni ottenute.

Scheda di ripasso sulle equazioni

..beginexe
  1. L’equazione \(25x ^{2 } + 1 = 0\) ha per soluzioni

    [A] \(x = \pm 5\) [B] \(x = \pm \frac{1 }{5 }\) [C] x=-5 e x=0 [D] non ha soluzioni reali

  2. L’equazione \(16x ^{2 } + x = 0\) ha per soluzioni

    [A] \(x = 4 \vee x = 1\) [B] \(x = \pm \frac{1 }{4 }\) [C] \(x = - \frac{1 }{16 } \vee x = 0\) [D] non ha soluzioni reali

  3. L’equazione \(4x ^{2 } - 9x = 0\) ha per soluzioni

    [A] \(x = \pm \frac{3 }{2 }\) [B] \(x = \pm \frac{9 }{4 }\) [C] \(x = \frac{3 }{2 } \vee x = 0\) [D] \(x = \frac{9 }{4 } \vee x = 0\)

  4. L’equazione \(9x ^{2 } + 6x + 1 = 0\) ha per soluzioni

    [A] \(x = \pm 3\) [B] \(x = \pm \frac{1 }{3 }\) [C] \(x = - \frac{1 }{3 }\) doppia [D] non ha soluzioni reali

  5. L’equazione \(x ^{2 } - 6x + 36 = 0\) ha per soluzioni

    [A] \(x = \pm 6\) [B] \(x = \pm \sqrt{6 }\) [C] \(x = 6\) doppia [D] non ha soluzioni reali

  6. Quale di queste equazioni ammette una soluzione doppia x=3?

    [A] \(2x ^{2 } - 12x + 18 = 0\) [B] \(9 - x ^{2 } = 0\) [C] \(x ^{2 } + 6x + 9 = 0\) [D] \(3x ^{2 } + 9x = 0\)

  7. Le soluzioni di un’equazione di secondo grado sono x1=1 e x2=3 . L’equazione è pertanto:

    [A] \(x ^{2 } + x - 1 = 0\) [B] \(x ^{2 } - 4x + 3 = 0\) [C] \(x ^{2 } - 4x - 3 = 0\) [D] \(x ^{2 } + 4x - 3 = 0\)

  8. Il polinomio \(x ^{2 } + 5x + 6\) può essere scomposto in:

    [A] \(( x + 2 ) ( x - 3 )\) [B] \(( x + 5 ) ( x + 1 )\) [C] \(( x - 2 ) ( x - 3 )\) [D] nessuna delle risposte precedenti

  9. Una delle soluzioni dell’equazione \(x ^{2 } - ( \sqrt{2 } + 1 ) x + \sqrt{2 } = 0\) è \(\sqrt{2 }\) , quanto vale l’altra?

    [A] \(- \sqrt{2 }\) [B] \(\frac{1 }{\sqrt{2 } }\) [C] \(\sqrt{2 } + 1\) [D] 1

  10. Per quale valore di k l’equazione \(( 2k - 1 ) x ^{2 } + ( 2k + 1 ) x + k - 2 = 0\) diventa di I grado?

    [A] \(k = \frac{1 }{2 }\) [B] \(k = - \frac{1 }{2 }\) [C] \(k = 2\) [D] \(k = 0\)

  11. L’equazione \(4m ^{2 } x ^{2 } - 5mx + 1 = 0\) con parametro m ha per soluzioni

    [A] \(x = m \vee x = 4m\) [B] \(x = \frac{1 }{m } \vee x = \frac{1 }{4m }\) [C] \(x = 64m \vee x = 1\) [D] \(x = m \vee x = \frac{1 }{4 }\)

  12. L’equazione di secondo grado \(x ^{2 } + ( a + 1 ) x + a = 0\) con a parametro reale ha come soluzioni:

    [A] \(x = 1 \vee x = a\) [B] \(x = a - 1 \vee x = 1\) [C] \(x = - a \vee x = - 1\) [D] \(x = a + 1 \vee x = a\)

  13. L’equazione \(x ^{2 } + ( t - 2 ) = 0\) con t parametro reale ammette soluzioni reali

    [A] per \(t \leq 2\) [B] per \(t \geq 2\) [C] per \(t < 2\) [D] nessuna delle risposte precedenti

  14. Quanto vale il prodotto delle soluzioni dell’equazione \(x ^{2 } - 6a ^{2 } x + 8a ^{4 } = 0\) ?

    [A] \(8a ^{4 }\) [B] \(8a ^{2 }\) [C] \(6a ^{2 }\) [D] non esiste

  15. Il polinomio \(x ^{2 } + ( m - 2 ) x - 2m\) con m parametro reale può essere scomposto in:

    [A] \(( x + m ) ( x + 1 )\) [B] \(( x + m ) ( x - 2 )\) [C] \(( x + m ) ( x + 2 )\) [D] \(( x - m ) ( x - 2 )\)

  16. L’equazione \(x ^{2 } + ( k - 1 ) x = 0\) con k parametro reale:

    [A] non ha soluzioni reali [B] ha una soluzione uguale a zero

    [C] ha due soluzioni reali coincidenti per k=0 [D] ha soluzioni reali e distinte per k=1

  17. L’equazione \(x ^{2 } + 2x + k - 2 = 0\) con k parametro reale:

    [A] ha due soluzioni reali coincidenti per k=3

    [B] ha due soluzioni reali coincidenti per k=1

    [C] ha una soluzione nulla per \(k = - 2\)

    [D] ha soluzioni reali e distinte per \(k \neq 3\)

  18. L’equazione \(x ^{2 } + m ^{2 } + 1 = 0\) con \(m\) parametro reale:

    [A] ammette due soluzioni reali e opposte [B] ammette due soluzioni coincidenti

    [C] non ammette soluzioni reali [D] ammette due soluzioni negative

  19. L’equazione \(2x ^{2 } + k ^{2 } = 0\) con \(k\) parametro reale:

    [A] ammette due soluzioni reali e distinte [B] ammette due soluzioni reali solo se \(k\) è positivo

    [C] ammette soluzioni coincidenti per \(k = 0\) [D] nessuna delle risposte precedenti è corretta

  20. L’equazione \(tx ^{2 } - 1 = 0\)

    [A] ha come soluzioni \(x _{1 } = 0 e x _{2 } = 1 - t\) [B] ammette sempre soluzioni reali

    [C] ammette soluzioni reali per \(t > 0\) [D] ha come soluzioni \(x = \pm t\)

Risposte

1.D-2.C-3.D-4.C-5.D-6.A-7.B-8.D-9.D-10.A-11.B-12.C-13.A-14.A-15.B-16.B-17.A-18.C -19.C-20.C

Copyright © Matematicamente.it 2011-2012

_images/immagini10.png

Questo libro, eccetto dove diversamente specificato, è rilasciato nei termini della licenza Creative Commons Attribuzione – Condividi allo stesso modo 3.0 Italia

il cui testo integrale è disponibile al sito

http://creativecommons.org/licenses/by-sa/3.0/it/legalcode

Tu sei libero:

di riprodurre, distribuire, comunicare al pubblico, esporre in pubblico, rappresentare, eseguire e recitare quest’opera, di modificare quest’opera, alle seguenti condizioni:

Attribuzione — Devi attribuire la paternità dell’opera nei modi indicati dall’autore o da chi ti ha dato l’opera in licenza e in modo tale da non suggerire che essi avallino te o il modo in cui tu usi l’opera.

Condividi allo stesso modo — Se alteri o trasformi quest’opera, o se la usi per crearne un’altra, puoi distribuire l’opera risultante solo con una licenza identica o equivalente a questa.

Autori

Erasmo Modica: teoria, esercizi

Anna Cristina Mocchetti: teoria, esercizi

Claudio Carboncini: coordinamento, editing

Antonio Bernardo: coordinamento, integrazioni, esercizi

Francesco Daddi: esercizi

Germano Pettarin: esercizi

Pierluigi Cunti: esercizi

Lisa Maccari: esercizi

Gemma Fiorito: correzioni

Sara Gobbato: integrazioni

Eugenio Medaglia: suggerimenti

Luciano Sarra: correzioni

Lucia Rapella: correzioni

Collaborazione, commenti e suggerimenti

Se vuoi contribuire anche tu alla stesura e aggiornamento del manuale Matematica C3 o se vuoi inviare dei commenti e/o suggerimenti scrivi a antoniobernardo@matematicamente.it

Versione del documento

Versione 2.1 del 21.06.2012